You are on page 1of 96

[J] Physics (Class XI) - Volume - III

CHAPTER - 01
GRAVITATION

Gravitation is the name given to the force of attraction between any two bodies in the universe. It was discovered
by Newton at the age of twentythree, in the year 1665.By comparing the acceleration due to gravity due to earth
with the acceleration required to keep the moon in its orbital around the earth, he was able to arrive the Basic Law
of Gravitation.
Gravitational force is the weakest force among the four fundamental forces of nature, but is the most important
force because it played an important role in initiating the birth of stars, and controlling the entire structure of the
universe.
NEWTON’S LAW OF GRAVITATION
Newton’s law of gravitation states that every body in this universe attracts every other body with a force,
which is directly proportional to the product of their masses and inversely proportional to the square of the distance
between their centres. The direction of the force is along the line joining the particles.
Thus the magnitude of the gravitational force F that two particles of masses m1 and m2 separated by a
m1m 2
distance r exert on each other is given by F 
r2
m1m 2
or F  G ------(1)
r2

VECTOR FORM OF NEWTON’S LAW OF GRAVITATION


Consider two bodies A and B of masses m1 and m2, placed at a distance r apart,
According to Newton’s law of gravitation
 Gm1m 2 Gm1m 2  Gm m 
F12  2
rˆ21  3
r21   13 2 r21
r r r21

r12 = unit vector from A to B.

r̂21 = unit vector from B to A.



F12 = gravitational force exerted on body A by body B.

F21 = gravitational force exerted on body B by body A
5
[J] Physics (Class XI) - Volume - III


Here negative sign indicates that the direction of F12 is opposite to that of r̂21 .

 Gm1m 2 Gm1m 2  Gm m 


Similarly F21  2
rˆ12  3
r12   1 3 2 r12 ------(2)
r r r12

Gm1m 2
 rˆ21  rˆ12   rˆ21 
r2
 
 It is clear that F12  F21 . Which is Newton’s third law of motion.
Here G is the constant of proportionality which is called ‘Universal gravitational constant.’
if m1 = m2 and r = 1 then G = F.
ie, universal gravitational constant is equal to the force of attraction between two bodies each of unit mass
whose centres are placed unit distance apart.
i) The value of G in the laboratory was first determined by Cavendish using the torsional balance.
ii) The value of G is 6.67 × 10–11 N-m2 kg–2 in S.I. and 6.67 × 10–8 dyne-cm3-g–2 in C.G.S. system.
iii) Dimensional formula [M–1 L3 T–2]
iv) The value of G does not depend upon the nature and size of the bodies.
v) It also does not depend upon the nature of the medium between the two bodies.
vi) As G is very small hence gravitational forces are very small, unless one (or both) of the masses is huge.
Properties of Gravitational Force
1. It is always attractive in nature while electric and magnetic forces can be attractive or repulsive.
2) It is independent of the medium between the particles while electric and magnetic force depend on the nature of
the medium between the particles.
3) It holds good over a wide range of distances. It is found true for interplanetary to inter atomic distances.
4) It is a central force, ie, acts along the line joining the centres of two interacting bodies.
5) It is a two-body interaction ie, gravitational force between two particles is independent of the presence or
absence of other particles; so the principle of superposition is valid ie, force on a particle due to number of
   
particles is the resultant of forces due to individual particles ie, F  F1  F2  F3  ............ , while nuclear force
is many body interaction.

6) It is the weakest force in nature : As Fnuclear  Felectromagnetic  Fgraviational .


7) The ratio of gravitational force to electrostatic force between two electrons is of the order of 10–43.
8) It is a conservative force ie, work done by it is path independent or work done in moving a particle round a
closed path under the action of gravitational force is zero.
9) It is an action reaction pair ie, the force with which one body (say earth) attracts the second body (say moon)
is equal to the force with which moon attracts the earth. This is in accordance with Newton’s third law of
motion.

6
[J] Physics (Class XI) - Volume - III

The law of gravitation is stated for two point masses, therefore for any two arbitrary finite size bodies, as shown
in the figure, it can not be applied as there is not a unique value for the seperation.

But if the two bodies are uniform spheres, then the separation r may be taken as the distance between their
centres because a sphere of uniform mass behaves as a point mass for any point lying outside it.
GRAVITY
Gravity is the force of attraction exerted by earth towards its centre on a body lying on or near the surface of
earth. Gravity is merely a special case of gravitation and is also called earth’s gravitational pull.
Force of gravity acting on a body is the measure of weight of the body. Thus weight of a body is defined as the
force of attraction exerted by the earth on the body towards its centre.
If m is the mass of a body placed on the surface of earth, where acceleration due to gravity is g, then, gravity pull
= weight of body = mg. The units and dimensions of gravity pull or weight are the same as those of force.
Example 1 : A mass M is split into two parts m and (M – m), which are then separated by a certain distance. What
ratio (m/M) maximises the gravitational force between the parts?
Solution : If r is the distance between m and (M – m), then gravitational force will be
m M  m G
FG 2
 2  mM  m 2 
r r
dF
For F to be maximum  0 as M and r are constants.
dm
d G 
2 
i.e.,  mM  m 2    0
dm  r 
 G 
i.e., M – 2m = 0 as r 2  0 

 m  1
or      , i.e., the force will be maximum when the two parts are equal.
M 2
Example 2 : Two particles of equal masses move in a circle of radius r under the action of their mutual gravitational
attraction. Find the speed of each particle if its mass is m.
Solution : The particles will always remain diametrically opposite so that the force on each particle will be directed
along the radius. Considering the circulation of one particle, we have

mv 2 Gmm Gm
 2 , i.e., v 
r  2r  4r
Example 3 : Three equal particles each of mass m are placed at the three corners of an equilateral triangle of side
a. Find the force exerted by this system on another particle of mass m placed at (a) the mid-point of a side, (b) at the
centre of the triangle.
Solution : As gravitational force is a two body interaction, the principle of superposition is valid, i.e., resultant force
on particle of mass m at P,
   
F  FA  FB  FC
7
[J] Physics (Class XI) - Volume - III

(a) As shown in figure, when P is at the mid-point of a side, FA and FB will be equal in magnitude but opposite in
direction so will cancel each other. So point mass m at P will experience a force due to C only, i.e.,

mm Gm 2 4Gm 2
F  FC  G   along PC
 CP   a sin 60 
2 o 2 3a 2

  
(b) When P is at the centre of the triangle, O, the forces of three particles FA , FB and FC will be equal in
magnitude and will subtend equal angles with each other (120o), so that the resultant force on m at O,
   
F  FA  FB  FC  0
Example 4 : A thin rod of mass M and length L is bent in a semicircle as shown in figure. (a) What is its gravitational
force (both magnitude and direction) on a particle with mass m at O, the centre of curvature? (b) What would be the
force on m if the rod is, in the form of a complete circle?
Solution : (a) Considering an element of rod of length dl as shown in figure. and treating it as a point of mass (M/L)dl
situated at a distance R from P, the gravitational force due to this element on the particle will be

Gm  M / L  Rd 
dF  along OP [as dl = R d ]
R2

So, the component of this force along x and y-axes will be


GmM cos d
dFH  dF cos  
LR
GmM sin d
dFV  dFsin  
LR

GmM GmM
So that Fx  LR  cos  d  LR sin 0  0

8
[J] Physics (Class XI) - Volume - III


GmM GmM
and Fy  LR  sin  d  LR   cos 0

2GmM  L

L2  as R   

2GmM
So, F  Fx  Fy  Fy 
2 2
[as Fx is zero]
L2

 2GmM 
i.e., the resultant force is along the y-axis and has magnitude  
 L2 
(b) If the rod was bent into a complete circle,
2
GmM
Fx 
LR  cos  d  0
0

2
GmM
and also Fy  LR  sin  d  0
0

i.e., the resultant force on m at O due to the ring is zero.


Example 5 : A point of mass is at a distance x from the centre of a ring of mass M and radius R on its axis as shown
in figure. Find the gravitational force between the two. What will this force be if x >> R and x << R ? For what value
of x is the force maximum?
Solution : Considering a small element of the ring (as shown in figure) and treating it as a point mass, by Newton’s
law of gravitation, force on the particle of mass m due to this element,

GmdM Gm  M / 2R  dl  M 
dF    as dM  2R dl 
r2 r2
Now due to symmetry of the problem the component of forces perpendicular to the axis due to all elements
will cancel each other, i.e.,
Fy   d Fy  0
while components along x-axis will add up so that
F   dF cos  along PO

GmM dl  x   x
2R  r 2  r 
i.e., F     as cos   r 

Now as for a given point and ring, x and r    R  x   will be the same for all elements,
2 2 1/ 2
 

GmMx GmMx
F
2R  R 2  x
 dl  as
 dl  2R 

2 3/2
R  x2 
3/2
2

9
[J] Physics (Class XI) - Volume - III

Now if x >> R, F   GmM / x  , then for a distant point ring behaves as a point mass (which is expected)
2

and for x << R, F   GmM / R  x , i.e., force varies linearly with distance x. Furthermore F will be maximum when
3

 dF 
 0,
 dx 

 
d  GmMx 
 0 or d  x  R 2  x 2 3/2   0 [as GmM  0]
R  x  
i.e., dx  2 2 3/2 
dx  

i.e.,  R  x   3x  R  x    0
2 2 3/2 2 2 2 5/2
 

or x  
R  as  R 2  x 2 5/2  0 
2  
Substituting this value of x in the expression for force F,

 2GMm 
Fmax   2 
3 3R 
Example 6 : A spherical hollow is made on a lead sphere of radius R such that its surface touches the outside
surface of the lead sphere and passes through its centre. The mass of the lead sphere before hollowing was M. What
is the force of attraction that this sphere would exert on a particle of mass m which lies at a distance d from the centre
of the lead sphere on the straight line joining the centres of the sphere and the hollow.
Solution : As the point mass m is outside the lead sphere we can assume its mass to be concentrated at the centre.
To calculate the force of attraction on the point mass m we should calculate the force due to the solid sphere and
subtract from this the force which the mass of the hollowed sphere would have exerted on m, i.e.,
GmM GmM
F 
d2 y2

10
[J] Physics (Class XI) - Volume - III

  R 
But from figure y  d    
  2 

4 3
3
4 R  M
and as M    R  , M          
3 3  2   8 

GMm Gm  M / 8  GMm  1 
So, F    1  
 d   R / 2   d 2  8 1   R / 2d   2 
2
d2
   
Acceleration Due to Gravity
The force of attraction exerted by the earth on a body is called gravitational pull or gravity.
We know that when force acts on a body, it produces acceleration. Therefore, a body under the effect of
gravitational pull must accelerate.
The acceleration produced in the motion of a body under the effect of gravity is called acceleration due to
gravity, it is denoted by g.
Consider a body of mass m, lying on the surface of earth then gravitational force on the body is given by
GMm
F ------(3)
R2
Where M = mass of the earth and R = radius of the earth.
If g is the acceleration due to gravity, then the force on the body due to earth is given by
Force = mass × acceleration.
or F = mg ------(4)

GMm
From (3) and (4) we have mg 
R2
GM
 g ------(5)
R2
G 4 3  4 3
 g  R   [As mass (M) volume  r   density    ]
R2  3  3 
4
 g GR ------(6)
3
Important Points
GM 4
i) From the expression g   GR it is clear that its value depends upon the mass radius and density of
R2 3
planet and it is independent of mass, shape and density of the body placed on the surface of the planet. ie, a
given planet (reference body) produces same acceleration in a light body as well as heavy body.

11
[J] Physics (Class XI) - Volume - III

ii) The greater the value of (M/R2) or R , greater will be value of g for that planet.
iii) Acceleration due to gravity is a vector quantity and its direction is always towards the centre of the planet.
iv) Dimension [g] = [LT–2]
v) It’s average value is taken to be 9.8 m/s2 or 981 cm/sec2 or 32 feet/sec2, on the surface of the earth at mean
sea level.
vi) The value of acceleration due to gravity vary due to the following factors: (a) Shape of the earth, (b) Height
above the earth surface, (c) Depth below the earth surface and (d) Axial rotation of the earth.
MASS OF EARTH
gR 2
From equation (5), we find M
------(7)
G
Substituting the values of G, g and R (6.4  106 m) we get M=6.018  1024 kg
DENSITY OF EARTH
Consider earth to be a spherical body of radius R. Let  be the uniform density of the material of the earth.
Mass M 3M
As density,     ------(8)
Volume 4 R 3 4R 3
3
From (7) and (8), we get
3g R 2 G 3g
  ------(9)
4R 3
4RG
We know that g = 9.8ms-2 ; R = 6.4  106 m; G = 6.67  10-111Nm2kg-2
From (9)
3  9.8
 = 5.4783  103 kgm-3  5.5  103 kgm-3
4  (22 / 7)  6.4  106  (6.67  1011 )
VARIATION OF ACCELERATION DUE TO GRAVITY
The value of acceleration due to gravity changes with height (i.e. altitude), depth, shape of the earth and rotation of
earth about its own axis.
Consider earth to be a sphere of mass M, radius R with centre at O. Let g be the value of acceleration due to gravity
at a point A on the surface of earth.,

GM
 g ------(10)
R2
If g' is the acceleration due to gravity at a point B, at a height h above the surface of the earth, then
GM
g 
R  h ------(11)
2

12
[J] Physics (Class XI) - Volume - III

Dividing (11) by (10), we get


g GM R2 R2
   
g  R  h 2 GM  R  h 2 --------(12)

2
R2  h
  1  
R 2 1  h R   R
2

If h<<R then h/R is very small as compared to 1. Expanding the R.H.S. of above equation by Binomial theorem and
neglecting the square and higher powers of h/R, we get
g 2h  2h 
 1 or g  g 1   --------(13)
g R  R
From (13), we note that the value of acceleration due to gravity decreases with height.
It is due to this reason that the value of acceleration due to gravity is lesser at mountains than in plains.
Important points
1
i) As we go above the surface of the earth, the value of g decreases because g  .
r2
ii) If r   then g' = 0, ie, at infinite distance from the earth, the value of g becomes zero.
iii) If h << R ie, height is negligible in comparison to the radius then from equation (12) we get
2 2
 R   h  2h 
g  g    g  1    g 1   [As h << R]
Rh  R  R
iv) If h<< R then decrease in the value of g with height:
2hg
Absolute decrease g  g  g 
R
g g  g 2h
Percentage decreases  100   100   100%
g g R
Variation in g with Depth
Consider earth to be a homogeneous sphere of radius R and mass M with centre at O. Let g be the value of
acceleration due to gravity at a point A on the surface of earth, (Fig. below)
GM
Then g
R2

A
d B

R
(R–d)
O

13
[J] Physics (Class XI) - Volume - III

4 3
If  is uniform density of material of earth, then M  R 
3
4
G  R 3
3 4
g 2
 GR ------(14)
R 3
Let g' be the acceleration due to gravity at the point B at a depth d below the surface of earth. The body at B will
experience gravity pull due to shaded portion of earth whose radius is (R – d) and mass is M'
GM
g  2 ------(15) and M  
4
  R  d   ------(16)
3

 R  d  3
Substituting (16) in (15)
4
G  R  d 
3
4
 g  3  G  R  d   ------(17)
R  d
2
3
Dividing (17) by (14), we get
4
G  R  d  
g 3 R d R d
   
g 4 R R R
GR
3

 d
or g  g  1   ------(18)
 R
From (18), we note that the value of acceleration due to gravity decreases with depth. At the centre of the earth,
d = R,
 R
g1=g0 (say). From (18), we get g 0  g  1    0 ------(19)
 R
It means the acceleration due to gravity is zero at the centre of earth. Therefore the weight of the body of mass
m at the centre of earth = mg0 = 0, but the mass of the body will not be zero.
Important Points
i) The value of g decreases on going below the surface of the earth. From equation (18) we get g   R  d  .
So it is clear that if d increases, the value of g' decreases.
ii) At the centre of earth d = R  g  0 , ie, the acceleration due to gravity at the centre of earth becomes zero.
iii) Decrease in the value of g with depth.
gd
Absolute decrease g  g  g 
R
g g  g d
Percentage decrease  100   100   100%
g R R
iv) The rate of decrease of gravity above the earth (if h << R) is double to that of inside the earth.

14
[J] Physics (Class XI) - Volume - III

Variation of g Due to Shape of Earth.


Earth is elliptical in shape. It is flattened at the poles and bulged out at the equator. The equatorial radius is
about 21 km longer than polar radius,
GM
from g  ------(20)
R2

GM
At equator g e  R 2 ------(21)
e

GM
At poles gp 
R 2p ------(22)

2
ge R p
From (20) and (21)  2
gp R e
2
Since R equator  R pole  g pole  g equator and g p  g e  0.018 ms
Example 7 : Compute the mass and density of the moon if acceleration due to gravity on its surface is 1.62 m/s2 and
its radius is 1.74 × 106 m (G = 6.67 × 10–11 MKS units)
 GM 
Solution : We know that g   2 
 R 

gR 2 1.62  1.74 10 


6 2

So, M    7.35  1022 kg


G 6.67 1011

M gR 2 3g
   
and as V G  4 / 3 R 3
4GR

3  1.62
So,    3.3  103 kg / m 3
4  3.14  6.67  10 11  1.74  106
Example 8 : Two equal masses m and m are hung from a balance whose scale pans differ in vertical height by h.
Calculate the error in weighing, if any, in terms of density of earth  .

15
[J] Physics (Class XI) - Volume - III


g  2h 
Solution : As with height ‘g’ varies as g   g 1  
1  h / R   R
2

and in accordance with figure h1 > h2, so W1 will be lesser than W2 and

h h 
W2  W1  mg 2  mg1  2mg  1  2 
R R 

 GM 
 as g  R 2 and  h1  h 2   h 
GM h
or W2  W1  2m
R2 R

2mhG  4 3  8  4 3 
or W2  W1   R    Gmh  as M  3 R  
R3  3  3

Variation in g due to Rotation of Earth


Effect of latitude (Due to Rotation of earth about its own axis)
Latitude at a place is defined as the angle which the line joining the place to the centre of the earth makes
with the equatorial plane. It is generally denoted by the letter  .

N
r P A
C


O E

S Equatorial
plane

In Fig., Latitude at a place P =  POE =  . As is evident from the figure,  = 90° at poles and  = 0° at
equator.
Consider earth to be a perfect sphere of mass M, radius R with centre O. The whole mass of earth can be
supposed to be concentrated at the centre O. As earth rotates about its polar axis from west to east, every particle
lying on its surface moves along a horizontal circle with same angular velocity as that of earth. The centre of each
circle lies on the polar axis.
Consider a particle of mass m at a place P of latitude  . If earth is rotating about its polar axis NS with
constant angular velocity  , then particle at P also rotates and describes a horizontal circle of radius r, where
r = PC = OP cos  = R cos 
The centrifugal force acting on the particle at P is m r  2. It acts along PA. directed away from the centre C of the
circle of rotation.)

16
[J] Physics (Class XI) - Volume - III

Z 

P Fc
C A
mg mg'
O  180 – 
B
Q' 
E
R

Z'
Suppose g is the acceleration due to gravity when earth were at rest, then the gravity pull on the particle (i.e.
true weight of the particle) = mg ; which acts along a vertical direction PO.
Let g' be the acceleration due to gravity at P when rotation of earth is taken into account.
Then apparent weight of the particle at
P = mg'
This is the resultant of the true weight and centrifugal force acting on the particle at P and hence must be
represented by the diagonal PB of parallelogram PABO.
Here  APO = (180° -  ).
Using parallelogram law of forces, we have

 mg    mr2   2  mg   mr2  cos 180   


2
mg 
2

g  g 2  r 2 4  2gr2 cos   g 2  R 2 cos 2 4  2gR cos 2 cos 


1
 R 2 4 2R2 2
 g 1  2 cos 2   cos 2   ------(23)
 g g 
2 2
We know that g = 9.8ms-2 and R=6.4  106m and   =  rad s -1 Then
T 24  60  60

R2 2
2
  1 1
 6.4  106     
g  24  60  60  9.8 298

R2
Since the value of is very small, therefore the terms with its squares and higher powers can be neglected
g
Hence (23) becomes
1
 2R2 cos 2   2
g  g  1  
 g 
Expanding it by Binomial theorem, we get
 1 2R2 cos 2    R2 cos 2  
g  g  1     g 1  
 2 g   g 

17
[J] Physics (Class XI) - Volume - III

or g  g  R2 cos 2  ------------(24)


As cos  and  are positive, thereforee g1  g
Thus acceleration due to gravity
(i) decreases on account of rotation of earth
(ii) increases with the increase in latitude of the place
Important points
i) Substituting   90o in the above expression we get g pole  g  2 R cos 2 90o

 g pole  g -------------(25)
ie, there is no effect of rotational motion of the earth on the value of g at the poles.
ii) Substituting   0o in the above expression we get g equator  g  2 R cos 2 0o

 g equator  g   R -------------(26)
2

ie, the effect of rotation of earth on the value of g at the equator is maximum.
From equation (25) and (26) g pole  g equator  R2  0.034 m / s 2
iii) When a body of mass m is moved from the equator to the poles, its weight increases by an amount
m  g p  g e   m2 R
iv) Weightlessness due to rotation of earth: As we know that apparent weight of the body decreases due to
rotation of earth. If  is the angular velocity of rotation of earth for which a body at the equator will become
weightless.
g  g  2 R cos 2 
 0  g  2 R cos 2 0o [As   0o for equator]

g
 g  2 R  0   ------(27)
R

2 R
or time period of rotation of earth T   2 --------(28)
 g
Substituting the value of R = 6400 × 103 m and g = 10 m/s2 we get
1 rad
  1.25  10 3 and T  5026.5 sec  1.40 hr
800 sec
1
Note: This is the time is abouttimes the present time period of earth. Therefore if earth starts rotating 17
17
times faster then all objects on equator will become weightless.
If earth stops rotation about is own then at the equator the value of g increases by 2 R
After considering the effect of rotation and elliptical shape of the earth, acceleration due to gravity at the poles
and equator are related as
g p  g e  0.034  0.018 m / s 2  g p  g e  0.052 m / s 2 ------(29)

18
[J] Physics (Class XI) - Volume - III

Inertial and Gravitational Masses


i) Inertial mass: It is the mass of the material body, which measures its inertia.
If an external force F acts on a body of mass mi, then according to Newton’s second law of motion

F
F  mi a or m i  .------(30)
a
Hence inertial mass of a body may be measured as the ratio of the magnitude of the external force applied on
it to the magnitude of acceleration produced in its motion.
Important points
i) It is the measure of ability of the body to oppose the production of acceleration in its motion by an external
force.
ii) Gravity has no effect on inertial mass of the body.
iii) It is proportional to the quantity of matter contained in the body.
iv) It is independent of size, shape and state of the body.
v) It does not depend on the temperature of body.
vi) It is conserved when two bodies combine physically or chemically.
vii) When a body moves with velocity v, its internal mass is given by

m0
m
v 2 , where m = rest mass of the body , c = velocity of light in vacuum,
1 o
c2
2) Gravitational Mass: It is the mass of the material body, which determines the gravitational pull acting upon it.
If M is the mass of the earth and R is the radius, then gravitational pull on a body of mass mg is given by

GMm g F F F
F or m g    ------(31)
R 2
 GM / R  g E
2

Here mg is the gravitational mass of the body, E is the gravitational intensity.


If at any point E = 1, then mg = F
Thus the gravitational mass of a body is defined as the gravitational pull experienced by the body in a gravitational
field of unit intensity.
3) Comparison between inertial and gravitational mass
i) Both are measured in the same units.
ii) Both are scalars.
iii) Both do not depends on the shape and state of the body.
iv) Inertial mass is measured by applying Newton’s second law of motion where as gravitational mass is
measured by applying Newton’s law of gravitation.
v) Spring balance measure gravitational mass and inertial balance measure inertial mass.

19
[J] Physics (Class XI) - Volume - III

4. Comparison between mass and weight of the body

Mass (m) Weight (W)


It is a quantity of matter contained in a body It is the attractive force exerted by earth
on any body
Its value does not change with g Its value changes with g
Its value can never be zero for any At infinity and at the centre of earth
material particle its value is zero
Its unit is kilogram and its dimension is [M] Its unit is Newton or kg-wt and
dimension are [MLT–2 ]
It is determined by a physcial balance It is determined by a spring balance
It is a scalar quantity It is a vector quantity
Gravitational Field
We know that earth attracts every body towards its centre. According to Newton’s law of gravitation, the
force of attraction varies inversely as the square of the distance of the body from the centre of earth. This shows that
in the space all around the earth, its gravitational pull can be experienced by other material bodies. The same is true
for every material body.
The space surrounding a material body in which gravitational force of attraction can be experienced is called
its gravitational field.
INTENSITY OF GRAVITATIONAL FIELD
The intensity of gravitational field at a point in a gravitational field is defined as the force experienced by
a body of unit mass placed at that point provided the presence of unit mass does not disturb the original
gravitational field. It is always directed towards the centre of gravity of the body whose gravitational field is
considered. Intensity of gravitational field at a point is a vector quantity and is denoted by E.

So if a test mass m at a point in a gravitational field experiences a force F then

 F
E  ------(32)
m
Important points
i) It is a vector quantity and is always directed towards the centre of gravity of body whose gravitational field
is considered.
ii) Units : Newton/kg or m/s2
iii) Dimension : [MoLT–2]
iv) If the field is produced by a point mass M and the test mass m is at a distance r from it then by Newton’s
GMm
law of gravitation F 
r2

F GMm / r 2
then intensity of gravitational field E  
m m
GM
 E
r2
20
[J] Physics (Class XI) - Volume - III

v) As the distance (r) of test mass from the point mass (M), increases, intensity of gravitational field decreases.
GM 1
E 2
;  E 2
r r
vi) Intensity of gravitational field E = 0, when r  
vii) Intensity at a given point (P) due to the combined effect of different point masses can be calculated by
vector sum of different intensities.
   
E net  E1  E 2  E3  .............

viii) Point of zero intensity: If two bodies A and B of different masses m1 and m2 are ‘d’ distance apart.
Let P be the point of zero intensity ie, the intensity at this point is equal and opposite due to two bodies A
and B and if any test mass placed at this point it will not experience any force.

  Gm1 Gm 2


For point P E1  E 2  0  x 2  d  x 2  0
 
m1 d m2 d
By solving x and  d  x   ------(33)
m1  m 2 m1  m 2
ix) Gravitational field line is a line, straight or curved such that a point mass placed in the field of another
mass would always move along this line. Field lines for an isolated mass m are radially inwards.

21
[J] Physics (Class XI) - Volume - III

GM GM
x) As E  2 and also g  2  E  g
r R
Thus the intensity of gravitational field at a point in the field is equal to acceleration of test mass placed at that
point.
Gravitational Potential
At a point in a gravitational field, potential V is defined as negative of work done per unit mass in shifting a test
mass from some reference point (usually at infinity) to the given point ie,
 
W F . dr    F 
V      E . dr  As m  E 
m m
dV GM
 E ------(34) , V
dr r
ie, negative gradient of potential gives intensity of field or potential is a scalar function of position whose space
derivative gives intensity. Negative sign indicates that the direction of intensity is in the direction where the potential
decreases.
a) Potential at a point on the surface of the solid sphere
If the point P lies on the surface of the solid sphere, we have r = R, the radius of the sphere.
M
So that, graviational potential at a point on the surface of a solid sphere =  G
R
b) Potential at a point inside the solid sphere
Let the point P now lie inside the solid sphere distance r from the centre O of the sphere. i.e., now r < R.
The solid sphere may be imagined to be made up of an inner solid sphere of radius r surrounded by a number
of spherical shells, concentric with it and with their radii ranging from r to R. The potential at P due to the whole solid
sphere is then clearly equal to the sum of the potentials at P due to the inner solid sphere and all the spherical shells
outside it.

Clearly, point P lies on the surface of the inner solid sphere of radius r and inside all the spherical shells of radii
greater that r. So that, potential at P due to the inner solid sphere of radius r.
mass of the sphere 4 4 4
 G   r 3G / r   r 2G , because masso f the inner solid sphere  r 2 ,
r 3 3 3
where  is the volume density of the sphere.
To determine the potential at P due to all the outer shells, let us consider one such shell of the x and thickness dx,
i.e., of volume = area × thickness = 4x 2 dx and hence of mass = 4x 2 dx . Since potential at a point inside a shell
is the same as that at a point on its surface, we have potential at P due to this shell

22
[J] Physics (Class XI) - Volume - III

4x 2 dx
 G  4xdxG
x
R

 Potential at P due to all the shells   4Gx dx


r

R
R
 x2 
 4G  x dx  4G  
r  2 r

 R 2  r2  4 3 R 2  r2 
 4G     G
 2  3 2

4
  G
 3R 2  3r 2 
3 2
 Potential at P due to the whole solid sphere = potential at P due to inner solid sphere + potential at P due to
all the spherical shells.

4 4  3R 2  3r 2 
  r 2G  G  
3 3  2 

4  2 3R 2 3r 2  4  3R 2  r 2 
  G  r      G  
3  2 2  3  2 

4  3R 2  r 2 
  R 3G  3  [Multiplying and dividing by R3]
3  2R 

4 3
Clearly, R  is the mass of the whole solid sphere i.e., M.
3
 Gravitational potential at P due to the solid sphere, i.e.,

M  3R 2  r 2 
V G
2R 3
Important points
i) It is a scalar quantity because it is defined as work done per unit mass.
ii) Unit : Joule / kg or m2/sec2
iii) Dimension : [MoL2T–2]

 GM   GM 
iv) If the field is produced by a point mass then V    E dr      2  dr  As E  
 r  r 2 

GM
 V c [Here c = constant of integration]
r
23
[J] Physics (Class XI) - Volume - III

Assuming reference point at  and potential to be zero there we get


GM
0 cc0

GM
 Gravitational potential V   ------(35)
r
v) Gravitational potential difference : It is defined as the work done to move a unit mass from one point to the
other in the gravitational field. The gravitational potential difference in bringing unit test mass m from point A to point
B under the gravitational influence of source mass M is

WA B 1 1
V  VB  VA   GM    ------(36)
m  rB rA 

vi) Potential due to large numbers of particle is given by scalar addition of all the potentials.
V  V1  V2  V3  ...........

GM GM GM
   .........
r1 r2 r3
in
Mi
 G  ------(37)
i 1 ri

vii) Point of zero potential: It is that point in the gravitational field, if the unit mass is shifted from infinity to that
point then net work done will be equal to zero. Let m1 and m2 are two masses placed at d distance apart and P is
the point of zero potential in between the two masses.
Net potential for point P  VA  VB  0

Gm1 Gm 2 m1d
   0 By solving x  m  m ------(38)
x dx 1 2

24
[J] Physics (Class XI) - Volume - III

Example 9: What are the values of gravitational attraction and potential at the surface of earth referred to zero
potential at infinite distance? Given that the mass of the earth is 6 × 1024 kg, the radius of earth is 6400 km and G =
6.67 × 10–11 MKS units.
Solution : According to the theory of gravitation, for an external point a spherical mass distribution behaves as if the
whole of its mass were concentrated at the centre; so
GM  6.67 10    6 10 
11 24

I   9.8 N / kg
R2  6.4 10 
6 2

GM  6.67 1011    6 1024 


and V   R    6.25  107 J / kg
 6.4 10 
6

Example 10 : The distance between earth and moon is about 3.8 × 105 km. At what point or points will be
gravitational field strength of earth-moon system be zero? Given mass of earth is 81 times the moon’s mass.
Solution: In this problem, earth and moon will be treated as point masses as they are far apart and as in case of a
M
point mass I  , the field due to both the masses will be zero at r   . In addition to it, the field will also be zero
r2
at a point in between earth and moon where the pull of earth balances the pull of moon. If this point is at a distance
x from the earth,
GM E GM M 81 1
 
 3.8 108  x  or x  3.8 108  x  [as M E  81M M ]
2 2 2 2
x

or 9  3.8  108  x   x
or x  3.42  108 m  3.42  105 km
Example 11 : The magnitudes of gravitational field at distances r1 and r2 from the centre of a uniform sphere of
 I1 
radius R and mass M are I1 and I2 respectively. Find the ratio of   if (a) r1 > R and r2 > R and (b) r1 < R and
 I2 
r2 < R.
GM
Solution : In case of spherical volume distribution of mass, I  for r > R
r2
GM
and I  r for r  R
R3
(a) As both r1 and r2 are greater than R,

I1  GM / r1  r22
2

 
I 2  GM / r22  r12

(b) As both r1 and r2 are lesser than R,

I1  GM / R  r1 r1
3

 
I 2  GM / R 3  r2 r2
25
[J] Physics (Class XI) - Volume - III

Example 12 : Two concentric shells of masses M1 and M2 are situated as shown in figure. Find the force on a
particle of mass m when the particle is located at (a) r = a (b), r = b and (c) r = c. The distance r measured from the
centre of the shell.

 GM 
Solution : We know that attraction at an external point due to a spherical shell of mass M is  2  while at an
 r 
internal point is zero. So:
(a) For r = a, the point is external to both the shells; so
G  M1  M 2 
IA 
a2

Gm  M1  M 2 
So, FA  mI A 
a2
(b) For r = b, the point is external to the shell of mass M2 and internal to the shell of mass M1; so

GM 2  GM 2 m 
IB   0 , i.e., FB  mI B   
 b 
2 2
b
(c) For r = c, the point is internal to both the shells; so
IC  0  0  0 , i.e., FC  mIC  0
Example 13 : A solid sphere of uniform density and radius 4 units is located with its centre of at the origin O of co-
ordinates. Two spheres of equal radii 1 unit with their centres at A (–2, 0, 0) and B(2, 0, 0) respectively are taken out
of the solid leaving behind spherical cavities as shown in figure. Then which of the following statements are true?
(a) The gravitational force due to this object at the origin is zero.
(b) The gravitational force at the point B(2, 0, 0) is zero.
(c) The gravitational potential is the same at all points on the circle y 2  z 2  36

(d) The gravitational potential is the same at all points on the circle y 2  z 2  4 .

26
[J] Physics (Class XI) - Volume - III

4 3
Solution : Considering the sphere to be made of spheres A and B of radius 1 unit and the remainder R, M    4 
3
4 3 M
and M A  M B    1     .
3  64 
    
(a) As the force at the centre of a sphere is zero FA  FB  FR  0 and FA and FB are equal and opposite for
  
point O, i.e. FA  FB  0 , so FR  0 , i.e., statement (a) is true.
 GMm 
(b) As the force at a distance r(< R) from the centre of a solid sphere. F    r , so F due to whole sphere
 R 
3

 GMm 
at B will be  3  2 . So
 4 
    2GMm 
FA  FB  FR   
 64 

However, FB = 0 as the point B at the centre of sphere B and FA   G  M / 64  m / 4  as the point B is


2

outside it. So
2GMm GMm GMm  1
FR     2   0
64 64  16 64  16 
i.e., statement (b) is not true.
(c) For a circle y2 + z2 = 36, the radius of the circle = 6 and so the point is outside the sphere and as for external
 GM 
point V    .
 r 
 GM 
VA  VB  VR    
 6 
Now as for spheres A and B the distance of the point on the cirlce r  62  22  40 and the point is
outside the spheres A and B, so
G(M / 64)
VA  VB 
40

GM  GM  GM  1 1 
 VR    2    
2  3 32 10 
 = constant.
6  64 40 
i.e., potential at all points of the circle y2 + z2 = 36 is same, so statement (c) is true.
(d) For a circle y2 + z2 = 4, the radius of the circle = 2 and so the point is inside the sphere at a distance (4 –2) =
2 units from its centre so

3 
3R 2  r 2   
GM GM 22GM
 V  48  4   
2R 2  64 64
27
[J] Physics (Class XI) - Volume - III

 22GM 
So VA  VB  VR    
 64 

But as for spheres A and B, the distance of the pont on the cirlce r  2 2  2 2  2 2 and the point is

  GM / 64 
outside the sphere A or B, so VA  VB 
2 2

 22GM GM  GM  1 
 VR    2    22 
2 
= constant.
 64 64  2 2  64 
i.e., potential at all points of the cirle y2 + z2 = 4 is same; so statement (d) is true.
Gravitational Potential Energy
Gravitational potential energy of a body at a point in a gravitational field of another body is defined as
the amount of work done in bringing the given body from infinity to that point without acceleration.
When the body is at infinity with respect to another body, the gravitational attraction on the body is zero.
Therefore its potential energy is zero, which is called zero level of potential energy.
The gravitational potential energy of a body at a point is defined as the amount of work done in bringing the
body from infinity to that point against the gravitational force.

r
1
r
GMm
W dx   GMm  
 x 
2

x
GMm
W
r
This work done is stored inside the body as its gravitational potential energy
GMm
U   ------(39)
r
Important points
i) Potential energy is a scalar quantity.
ii) Unit : Joule
iii) Dimensions : [ML2T–2]
iv) Gravitational potential energy is always negative in the gravitational field because the force is always attractive
in nature.
v) As the distance r increases, the gravitational potential energy becomes less negative ie, it increases.
vi) If r   then it becomes zero (maximum)
vii) In case of discrete distribution of masses.

 Gm1m 2 Gm 2 m3 
Gravitational potential energy U   u i      .........
 r12 r23 
28
[J] Physics (Class XI) - Volume - III

viii) If the body of mass m is moved from a point at a distance r1 to a point at distance r2 (r1 > r2) then change in
potential energy
1 1 1 1 
r2
GMm
U   2
dx   GMm    or U  GMm    ------(40)
r1
x  r2 r1   r1 r2 
As r1 is greater than r2, the change in potential energy of the body will be negative. It means that if a body is
brought closer to earth it’s potential energy decreases.
GMm  GM 
ix) Relation between gravitational potential energy and potential U    m
r  r 
 U = mV
x) Gravitational potential energy at the centre of earth relative to infinity.
 3 GM  3 GMm
U centre  mVcentre  m    ------(41)
 2 R  2 R
xi) Gravitational potential energy of a body at height h from the earth surface is given by

GMm gR 2 m mgR
Uh     ------(42)
Rh Rh 1
h
R
Work Done Against Gravity
If the body of mass m is moved from the surface of earth to a point at distance h above the surface of earth,
then change in potential energy or work done against gravity will be
1 1 
W  U  GMm   
 r1 r2 
1 1 
 W  GMm  
 R R  h 
[As r1 = R and r2 = R + h]

GMmh mgh
W  GM
  h h  g]
R 2 1   1  [As
R2
 R R
Important points
i) When the distance h is not negligible and is comparable to radius of the earth, then we will use above formula.

 n 
ii) If h = nR then W  mgR   ------(43)
 n 1

1
iii) If h = R then W  mgR
2
iv) If h is very small as compared to radius of the earth then term h/R can be neglected.

mgh  h 
From W 
1 h / R
 mgh -------------(44)  As R  0 

29
[J] Physics (Class XI) - Volume - III

ESCAPE VELOCITY
When a body is projected vertically upwards from the surface of the earth, with a certain velocity, its motion is
opposed by gravitational pull and resistance of earth’s atmosphere, due to which the body rises up to a certain height
and then falls back to the earth. As velocity of projection is increased, the body attains a greater height before falling.
Finally a stage reaches when the velocity is so large that it just crosses the gravitational field and will never return
back to the earth on its own. The body is said to have escaped. The minimum velocity of projection to achieve this
stage is called escape velocity.
Thus escape velocity of a body is defined as the minimum velocity with which the body has to be projected
vertically upwards from the surface of the earth (or any other planet) so that it just crosses the gravitational
field of the earth (or of that planet) and never returns on its own.

Q
dx
P

x A

R
O

Let earth be a perfect sphere of mass M, radius R with centre at O. Let a body of mass m to be projected from
a point A on the surface of earth, (Fig. above). Join OA and produce it further.
Take two points P and Q at a distance x and (x + dx) from the centre O of the earth.Gravitational force of

attraction on the body at P is F  GMm


x2
This much force has to be applied on the body in the upward direction. Work done in taking the body against
GMm dx
gravitational attraction from P to Q is dW  Fdx 
x2
Total work done in taking the body against gravitational attraction from surface of earth (i.e x = R) to a region
beyond the gravitational field of earth (i.e. x   ) can be calculated by integrating the above expression within the
limits x = R to x   . Thus total work done is
 
GMm   x 1  1 

W dx  GMm R x 2
dx  GMm  1    GMm
R
x2  R  x  R


1 1 GMm
 GMm     ------(45)
  R R R
This work done is at the cost of kinetic energy given to the body at the surface of the earth. If ve is the escape
velocity of the body projected from the surface of earth, then
1 2
Kinetic energy of the body  2 mv e ------(46)

30
[J] Physics (Class XI) - Volume - III

From (45) and (46)

 1 mv 2e  GMm or v 2e  2GM or v e  2GM ------(47)


2 R R R

As g  GM2
 GM  gR 2
R
2gR 2
Putting this value in (31), we get v e   2gR ------------(48)
R
4 R 3
If  is the mean density of the material of earth then, M 
3
8GR 2
Putting this value in (47), we get v e  2G  4 R 3  ------(49)
R 3 3
Important points
i) Escape velocity is independent of the mass and direction of projection of the body.
ii) Escape velocity depends on the reference body. Greater the value of (M/R) or (gR) for a planet, greater
will be velocity.
iii) For the earth as g = 9.8 m/s2 and R = 6400 km

 v e  2  9.8  6.4  106  11.2 km / sec

 3RT 
iv) A planet will have atmosphere if the velocity of molecule in its atmosphere  v rms  M  is lesser than
 
escape velocity. This is why earth has atmosphere (as at earth v rms  v e ) while moon has no atmosphere
(as at moon v rms  v e ).
v) If body projected with velocity lesser than escape velocity (v < ve) it will reach a certain maximum height
and then may either move in an orbit around the planet or may fall down back to the planet.
Example 14 : Three particles each of mass m are placed at the corners of an equilateral triangle of side d. Calculate
(a) the potential energy of the system, (b) work done on this system if the side of the triangle is changed from d to 2d.

 Gm1m 2 
Solution : (a) As in case of two-particle system potential energy is given by    , so
 r 

U A  U12  U 23  U 31

Gmm 3Gm 2
So, U A  3 
d d

31
[J] Physics (Class XI) - Volume - III

3Gm 2
(b) When d is changed to 2d, U B  
2d

3Gm 2
So, work done  U B  U A 
2d

Example 15 : What will be the acceleration due to gravity on the surface of the moon if its radius were 1/ 4  th the

radius of earth and its mass 1/ 80  th the mass of earth? What will be the escape velocity on the surface of moon
if it is 11.2 km/s on the surface of the earth? (given that g = 9.8 m/s2)

 GM 
Solution : As on the surface of planet g   2 
 R 
2
gM MM  R E  1 1
      4 
2

g E M E  R M  80 5

g E 9.8
 gM    1.96 m / s 2
5 5

Furthermore as escape velocity ve   2GM / R 

vM MM R E 1 1
So,    4 
vE ME R M 80 20

 v E   11.2 
i.e., v M      2.5 km / s
 20   4.47 
Example 16 : The masses and radii of the earth and moon are M1, R1 and M2, R2 respectively. Their centres are at
distance d apart. What is the minimum speed with which a particle of mass m should be projected from a point
midway between the two centres so as to escape to infinity?
Solution : Potential energy of m when it is midway between M1 and M2.

 GM1 GM 2  2Gm


U  m  V1  V2   m      M1  M 2 
 d/2 d / 2  d
And as potential energy at infinity is zero, so work required to shift m from the given position to infinity,

W  0  U  2Gm  M1  M 2  / d
As this work is provided by initial kinetic energy,

1 2Gm  M1  M 2  G  M1  M 2 
mv 2  or v  2
2 d d

32
[J] Physics (Class XI) - Volume - III

Example 17 : A mass 6 × 1024 kg (= mass of earth) is to be compressed in a sphere in such a way that the escape
velocity from its surface is 3 × 108 m/s (equal to that of light). What should be the radius of the sphere?

Solution : As ve   2GM / R  , R   2GM / v e2 

2  6.67 1011  6  1024


 R  9  103 m  9 mm
 3 10 
8 2

Example 18 : A particle is fired vertically upwards from the surface of earth and reaches a height 6400 km. Find the
initial velocity of the particle if R = 6400 km and g at the surface of earth is 10 m/s2.
Solution : If a body is projected from the surface of earth with a velocity v and reaches a height h, by conservation
of energy (relative to surface of earth).

1 mgh
mv 2 
2 1   h / R  

Here h = R = 6400 km and g = 10 m/s2

So, v 2  gh , i.e., v  10  6400 103  8 km / s

Example 19 : Imagine a planet whose diameter and mass are both one half of those of earth. The day’s temperature
of this planet surface reaches upto 800 K. Make calculation and tell whether oxygen molecules are possible in the
atmosphere of the planet. (Escape velocity from earth’s surface = 11.2 km/s, k = 1.38 × 10–23 J/K, mass of oxygen
molecule = 5.3 × 10–26 kg)

2GM
Solution : As ve 
R

vP MP R E 1 2
So,     1
vE ME R P 2 1

i.e., v P  v E  11.2 km / s ------(i)

Now according to kinetic theory of gases,

3RT 3  Nk  T
v rms   [as R = Nk and M = Nm]
M Nm

3kT 3 1.38 1023  800


i.e., v rms    0.79 km / s ----------(ii)
m 5.3 1026
From Eqns. (i) and (ii) it is clear that velocity of oxygen molecule in the atmosphere of planet (~ 0.8 km/s) is
much lesser than escape velocity from the planet (= 11.2 km/s); so oxygen molecules cannot escape and so the
planet’s atmosphere may contain oxygen.
33
[J] Physics (Class XI) - Volume - III

Example 20 : Distance between the centres of two stars is 10 a. The masses of these stars are M and 16 M and
their radii a and 2a, respectively. A body of mass m is fired straight from the surface of the larger star towards the
smaller star. What should be its minimum initial speed to reach the surface of the smaller star? Obtain the expression
in terms of G, M and a.

Solution : The distance (from the smaller planet) where the gravitational pull of two planet’s balance each other will
be given by

GMm G 16M  m

10a  x 
2
x2

i.e., x = 2 a
So, the body will reach the smaller planet due to planet’s gravitational field if it has sufficient energy to cross
the point B (x = 2a), i.e.,
1
mv 2  m  VB  VS 
2

16GM GM  65GM
but VS    2a  10a  2a    8a
  

16GM GM  20GM
and VB      
 8a 2a  8a

1  65GM 20GM 
mv 2  m  
8a 
So,
2  8a

3 5GM
i.e., v min 
2 a

Example 21 : A projectile is fired vertically upward from the surface of earth with a velocity Kve where ve is the
escape velocity and K < 1. Neglecting air resistance show that the maximum height to which it will rise measured
R
from the centre of earth is 1  K 2  where R is the radius of the earth.

Solution : If a body is projected from the surface of earth with velocity v and reaches a height h, by conservation of
energy (relative to surface of earth),

1 mgh
i.e., 2 mv  1   h / R  
2

 
34
[J] Physics (Class XI) - Volume - III

In this problem v  Kve  K 2gR and h   r  R 

1 mg  r  R  r  R 
mK 2
2gR  K2 
So, 2 1   r  R  / R  , i.e.,
r

R
or 1  K  r  R , i.e., r  1  K 2 
2

Example 22 : A rocket is fired with a speed v  2 gR near the earth’s surface and directed upwards. (a) Show

that it will escape from the earth. (b) Show that in interstellar space its speed is v  2gR

 GMm 
Solution : (a) As PE of the rocket at the surface of earth is    and at  , zero, so energy required for
 R 
escaping from earth.

 GMm   GM 
 0   mgR  as g  R 2 
 R 

1
And as initial KE of the rocket mv 2  2mgR is greater than the energy required for escaping ( = mgR), the
2
rocket will escape.
(b) If v is the velocity of the rocket in interstellar space (free from gravitational effects) then by conservation of
energy,
1
  1
  1
2 2
m 2 gR  m 2gR  mv 2
2 2 2

i.e., v 2  4gR  2gR

or v  2gR
vi) Maximum height attained by a body : Let a projection velocity of body (mass m ) is v, so that it attains
a maximum height h. At maximum height, the velocity of particle is zero, so kinetic energy is zero.
By the law of conservation of energy
Total energy at surface = Total energy at height h.
GMm 1 GMm
   mv 2   0
R 2 Rh

v2 1 1  GMh
  GM    
2  R R  h  R R  h

2GM R  h R
 2
  1
vR h h
35
[J] Physics (Class XI) - Volume - III

R R  v2  2GM 2GM
 h  2 R 2 2  [As ve    ve2 ]
 2GM  v e
   ve  v  R R
 2 1  1
 v R  v
2

 v2 
ie, h  R  2 2 -------(50)
 ve  v 

vii) If a body is projected with velocity greater than escape velocity  v  ve  then by conservation of
energy.
Total energy at surface = total energy at infinite
1 Gmm 1
mv 2   m  v   0
2

2 R 2

2GM  2GM 
ie,  v   v   v2  v 2  v e2  v e2 
2 2
 As
R R 

 v  v 2  ve2 -------(51)

ie, the body will move in interplanetary or inter setellar space with velocity v2  ve2 .
viii) Energy to be given to a stationary object on the surface of earth so that its total energy becomes zero,
is called escape energy.
GMm
Total energy at the surface of the earth = KE  PE  0 
R
GMm
 Escape energy 
R
ix) If the escape velocity of a body is equal to the velocity of light then from such bodies nothing can
escape, not even light. Such bodies are called black holes.

2GM  2 2GM 
The radius of a black hole is given as R
C2
-------(52) C  R 

GM
[As C  , where C is the velocity of light]
R

Kepler’s Laws of Planetary Motion


Planets are large natural bodies rotating around a star in definite orbits. The planetary system of the star sun
called solar system consists of nine planets, viz., Mercury, Venus, Earth, Mars, Jupiter, Saturn, Uranus, Neptune and
Pluto. Out of these planets Mercury is the smallest, closest to the sun and so hottest. Jupiter is largest and has
maximum moons (12). Venus is closest to Earth and brightest. Johannes Kepler, a German mathematician and
astronomer (1571-1630) after a life time study work out three empirical laws which govern the motion of these
planets and are known as Kepler’s laws of planetary motion. These are,

36
[J] Physics (Class XI) - Volume - III

(1) The law of Orbits : Every planet moves around the sun in an elliptical orbit with sun at one of the foci.

(2) The law of Area : The line joining the sun to the planet sweeps out equal areas in equal interval of time.i.e.
areal velocity is constant. According to this law planet will move slowly when it is farthest from sun and more rapidly
when it is nearest to sun. It is similar to the law of conservation of angular momentum
dA 1 r  vdt  1
Areal velocity    rv
dt 2 dt 2
dA L L
  ----------(53) [ As L = mvr; rv  ]
dt 2m m
(3) The law of periods: The square of period of revolution (T) of any planet around sun is directly proportional
to the cube of the semi-major axis of the orbit.
r r 
3

T  a or T   1 2 
2 3 2

 2 

From the figure AB = AF + FB


r r
2a  r1  r2  a  1 2 where a = semi - major axis.
2
r1 = Shortest distance of planet from sun (perigee)
r2 = Largest distance of planet from sun (apogee)
Important data

Kepler’s laws are valid for satellites also.


37
[J] Physics (Class XI) - Volume - III

Velocity of a Planet in Terms of Eccentricity


Applying the law of conservation of angular momentum at perigee and apogee

mv p rp  mv a ra

vp ra a  c 1  e c
    [As rp  a  c, ra  a  c and eccentricity e 
va rp a  c 1  e -------(54) a
]

Applying the conservation of mechanical energy at perigee and apogee

1 GMm 1 GMm 1 1
mv 2p   mv a2   v 2p  v a2  2GM   
2 rp 2 ra  rp ra 

 ra2  rp2   ra  rp   v 0 ra 
 v  2   2GM 
2
a   As v p  
 rp   ra rp   rp 

2GM  rp 
 va2   
ra  rp  ra 

2GM  a  c  2GM  1  e 
 v a2     
a ac a  1 e 

Thus the speeds of planet at apogee and perigee are

2GM  1  e  2GM  1  e 
va   , vp    ----------(55)
a  1 e  a  1 e 

Note: The gravitational force is a central force so torque on planet relative to sun is always zero, hence angular
momentum of a planet or satellite is always constant irrespective of shape of orbit
Some Properties of the Planet

38
[J] Physics (Class XI) - Volume - III

SATELLITE
A satellite is a body which is revolving continuously in an orbit around a comparatively much larger
body. For example, earth is a satellite of sun and moon in turn is a satellite of earth. In our solar system, each planet
is a satellite of sun, but some of these planets have got satellites around them. These are called natural satellites.
With the advancement of science and technology, since 1957, many man made satellites have been put in different
orbits around the earth. Such satellites are called Artificial satellites. Russians were the first to launch an artificial
satellite Sputnik I. on October 4. 1957. India launched its first artificial satellite — Aryabhatta in 1975. Since then,
India has put many satellites in various orbits around earth.
PRINCIPLE OF LAUNCHING A SATELLITE

Satellite
v

A1
A2
R A3

Earth

Consider a very high tower on the surface of earth whose height is more than the height of the earth’s atmosphere.
Let a body be projected from top of this tower with some horizontal velocity. The body will describe a parabolic
path under the effect of gravity and hit the surface of earth at point A1. If the initial horizontal velocity of projection of
the body is increased, the body will hit the surface of earth at point A2, farther from the foot of the tower, (Fig.
above). The horizontal range in second case is more than that in the first case.
If we go on increasing the initial horizontal velocity of projection of the body, its horizontal range will also increase
and finally a stage will come when the body will describe a stable circular path around the earth and becomes a
satellite of earth. The velocity required to put the satellite into its orbit around the earth is called orbital
velocity of the satellite.
Orbital Velocity of Satellite
Orbital velocity of a satellite is the velocity required to put the satellite into its orbit around the earth.
For revolution of satellite around the earth, the gravitational pull provides the requied centripetal force.

39
[J] Physics (Class XI) - Volume - III

mv 2 GMm GM
 ;  v
r r2 r
gR 2 g
v R ----------(56) [As GM = gR2 and r = R + h]
Rh Rh
Important points
i) Orbital velocity is independent of the mass of the orbiting body and is always along the tangent of the orbit i.e.,
satellites of different masses have same orbital velocity, if they are in the same orbit.
ii) Orbital velocity depends on the mass of central body and radius of orbit.

iii) 
For a given planet, greater the radius of orbit, lesser will be the orbital velocity of the satelite v  1/ r 
iv) Orbital velocity of the satellite when it revolves very close to the surface of the planet

GM GM GM
v   v  gR ----------(57) [As h = 0 and GM = gR2]
r Rh R

For the earth v  9.8  6.4  106  7.9 km / s  8 km / sec

GM  2GM 
v) Close to the surface of planet v  As v e  
R  R 
ve
 v i.e., vescape  2 v orbital ----------(58)
2
It means that if the speed of a satellite orbiting close to the earth is made 2 times (or increased by 41%) then
it will escape from the graviational field.
1
vi) If the gravitational force of attraction of the sun on the planet varies as F  then the orbital velocity varies
rn
1
as v  .
r n 1
Time Period of Satellite
It is the time taken by satellite to go once around the earth.
Circumference of the orbit
 T orbital velocity

2r r  GM 
 T  2r  As v  
v GM  r 

r3 r3
 T  2  2 [As GM = gR2]
GM gR 2

R  h
3 3/2
R  h
 T  2  2 1   ----------(59) [As r = R + h]
gR 2 g  R
40
[J] Physics (Class XI) - Volume - III

Important points
r3
i) From T  2 it is clear that time period is independent of the mass of orbiting body and depends on the
GM
mass of central body and radius of the orbit.

r3 4 2 3
ii) T  2 T 
2
r ie, T 2  r 3
GM GM
This is in accordance with Kepler’s third law of planetary motion r becomes a (semi major axis) if the orbit is
elliptic.
iii) Time period of nearby satellite,
r3 R3 R
From T  2  2 2
 2 ----------(60) [As h = 0 and GM = gR2]
GM gR g
For earth R = 6400 km and g = 9.8 m/s2
T = 84.6 minute  1.4 hr..
iv) Time period of nearby satellite in terms of density of planet can be given as

2  R 3 
1/2
r3 R3 3
T  2  2  
GM GM  4 3  1/2
G ----------(61)
G. 3 R  

1
v) If the gravitational force of attraction of the sun on the planet varies as F  then the time period varies as
rn
n 1
Tr 2 .
vi) If there is a satellite in the equatorial plane rotating in the direction of earth’s rotation from west to east, then for
an observer, on the earth, angular velocity of satellite will be  S  E  . The time interval between the two consecutive
appearances overhead will be
2 TT  2 
T  S E ----------(62)  As T  
S  E TE  TS  

If S  E , T   ie, satellite will appear stationary relative to earth. Such satellites are called geostationary
satellites.
Height of Satellite

R  h
3
r3
As we know, time period of satellite T  2  2
GM gR 2

gR 2 T 2
 R  h
3
By squaring and rearranging both sides
4 2

41
[J] Physics (Class XI) - Volume - III

1/3
 T 2 gR 2 
 h  2 
 R ----------(63)
 4 
By knowing the value of time period we can calculate the height of satellite from the surface of the earth.
Geostationary Satellite
The satellite which appears stationary relative to earth is called geostationary or geosynchronous satellite. It is
used as a communication satellite. A geostationary satellite always stays over the same place above the earth, and
such a satellite is never at rest. Such a satellite appears stationary due to its zero relative velocity with respect to that
place on earth.
The orbit of a geostationary satellite is known as the parking orbit
Important points
i) It should revolve in an orbit concentric and coplanar with the equatorial plane.
ii) Its sense of rotation should be same as that of earth about its own axis ie, in anti-clockwise direction (from west
to east).
iii) Its period of revolution around the earth should be same as that of earth about its own axis.
 T  24hr  86400 sec
iv) Height of geostationary satellite:
 R  h   24hr
3
r3
As T  2  2
GM GM
Susbstituting the value of G and M we get R + h = r  42000 km  7 R
 height of geostationary satellite from the surface of earth h  6R  36000 km.
GM
v) Orbital velocity of geostationary satellite can be calculated by v 
r
Substituting the value of G and M we get v = 3.08 km/sec.
Angular Momentum of Satellite
Angular momentum of satellite L = mvr
GM  GM 
 Lm r  As v  
r  r 

 L  m 2GMr -------(64)
ie, Angular momentum of satellite depend on both the mass of orbiting body and central body as well as the
radius of orbit.
Example 23 : A spaceship is launched into a circular orbit close to the earth’s surface. What additional velocity
should now be imparted to the spaceship in the orbit to overcome the gravitational pull. (Radius of earth = 6400 km
and g = 9.8 m/s2)
Solution : For orbiting the earth close to its surface,
mv 2 GMm GM  GM 
 2 , i.e., v0 
R
 gR  as g  R 2 
R R

i.e., v0   9.8  6.4 10   8 km / s


6

and for escaping from close to the surface of earth,

42
[J] Physics (Class XI) - Volume - III

GMm 1 2GM  GM 
R
 mve2 , i.e., ve 
2 R
 2gR  as g  R 2 

i.e., ve  2  v0  1.41 8 km / s  11.2 km / s


So, additional velocity to be imparted to the orbiting satellite for escaping = 11.2 – 8 = 3.2 km/s.
Example 24 : An artificial satellite is moving in a circular orbit around the earth with a speed equal to half the
magnitude of escape velocity from the earth. (a) Determine the height of the satellite above the earth’s surface. (b) If
the satellite is stopped suddenly in its orbit and allowed to fall freely into the earth, find the speed with which it hits the
surface of the earth. (g = 9.8 ms–2 and RE = 6400 km)
Solution : (a) We know that for satellite motion

v0 
GM
R
g  GM 
r R  h  as g  R 2 and r  R  h 

1 1
In this problem v 0  ve  2gR [as ve  2gR ]
2 2

R 2g 1
So,  gR ; i.e., 2R = h + R or h = R = 6400 km
Rh 2
(b) By conservation of ME
 Gmm  1  GMm 
0   mv   
2

 r  2  R 
1 1 
or v  2GM  
2

 R 2R 
[as r = R + h = R + R = 2R]

GM
or v   gR  10  6.4  106  8 km / s
R
Important points
i) In case of satellite motion, force is central so torque = 0, and hence angular momentum of satellite is consrved
ie, L = constant.
ii) In case of satellite motion as areal velocity
dA 1  r  vdt  1
  rv
dt 2 dt 2
dA L
  -------(65) [As L = mvr]
dt 2m

But as L = constant,  areal velocity (dA/dt) = constant which is Kepler’s II law..


ie, Kepler’s II law or constancy of areal velocity is a consequence of conservation of angualr momentum.
43
[J] Physics (Class XI) - Volume - III

Energy of Satellite
When a satellite revolves around a planet in its orbit, it possesses both potential energy (due to its position
against gravitational pull of earth) and kinetic energy (due to orbital motion).

GMm  L2  GM 
Potential energy : U  mV   2  As V  r , L  m GMr 
2 2
1)
r mr

1 GMm L2  GM 
2) Kinetic energy : K  mv 2    As v  
2 2r 2mr 2  r 

GMm GMm GMm L2


3) Total energy: E  U  K     ----------(66)
r 2r 2r 2mr 2
Important points
i) Kinetic energy, potential energy or total energy of a satellite depends on the mass of the satellite and the central
body and also on the radius of the orbit.
ii) From the above exprssions we can say that
Kineitc energy (K) = – (Total energy)
Potential energy (U) = 2 (Total energy)
Potential energy (K) = –2 (Kinetic energy)
iii) Energy graph for a satellite
iv) Energy distribution in elliptical orbit

v) If the orbit of a satellite is elliptic then

GMm
(a) Total energy (E) =  constant ; where a is a semi-major axis.
2a
(b) Kinetic energy (K) will be maximum when the satellite is closest to the central body (at perigee) and
maximum when it is farthest from the central body (at apogee)
(c) Potential energy (U) will be minimum when kinetic energy = maximum ie, the satellite is closest to the
central body (at perigee) and maximum when kinetic energy = minimum ie, the satellite is farthest from the
central body (at apogee).

44
[J] Physics (Class XI) - Volume - III

(vi) Binding Energy: Total energy of a satellite in its orbit is negative. Negative energy means that the satellite is
bound to the central body by an attractive force and energy must be supplied to remove it from the orbit to
infinity. The energy required to remove the satellite from its orbit to infinity is called Binding Energy of the
system, ie,
GMm
Binding Energy (B.E.) =  E  ----------(67)
2r
Change in the orbit of Satellite
When a satellite is transferred to a higher orbit (r2 > r1) then variation in different quantities can be shown by the
following table.
Quantities Variation Relation with r
Orbital velocity Decreases 1
v
r
Time period Increases T  r 3/2
Linear momentum Decreases 1
P
r
Angular momentum Increases L r
Kinetic energy Decreases 1
K
r
Potential energy Increases 1
U
r
Total energy Increases 1
E
r
Binding energy Decreases 1
BE 
r

Note: Work done in changing the orbit W = E2 – E1

 GMm   GMm 
W    
 2r2   2r1 
GMm  1 1 
W    ----------(68)
2  r1 r2 
Example 25 : Imagin a light planet revolving around a very massive star in a circular orbit of radius r with a period
of revolution T. On what power of r, will the square of time period depend if the gravitational force of attraction
between the planet and the star is proportional to r–5/2 ?

45
[J] Physics (Class XI) - Volume - III

Solution : As gravitation provides centripetal force


mv 2 K K
 5/2 , i.e., v  3/2
2

r r mr

2r mr 3/2
So that T   2r
v K

42 m 7/2
i.e., T 
2
r ; So T 2  r 7/2
K
Example 26 : Halley’s comet has a period of 76 years and in 1986, had a distance of closest approach to the sun
equal to 8.9 × 1010 m. What is the comet’s farthest distance from the sun if the mass of sun is 2 × 1030 kg and
G  6.67  1011 MKS units?
Solution : From the problem it is self-evident that the orbit of the comet is elliptic with sun being at one focus. Now
as for elliptic orbits, according to Kepler’s third law,

1/3
4 2 3  T 2 GM 
T 
2
a , i.e., a   2 
GM  4 

  76  3.15  107 2  6.67  10 11  2  1030 


1/3

or a     2.7  1012 m
 4 2 
 
But in case of ellipse,
2a  rmin  rmax , i.e., rmax  2a  rmin

So, rmax  2  2.7  1012  8.9  1019  5.3  1012 m


Example 27 : A sky lab of mass 2 × 103 kg is first launched from the surface of earth in a circular orbit of radius 2R
(from the centre of earth) and then it is shifted from this circular orbit to another circular orbit of radius 3 R. Calculate
the minimum energy required (a) to place the lab in the first orbit, (b) to shift the lab from first orbit to the second
orbit. Given, R = 6400 km and g = 10 m/s2.
Solution : The energy of the sky lab on the surface of earth

 GMm  GMm
E S  KE  PE  0    
 R  R
And the energy of the sky lab in an orbit of radius r

46
[J] Physics (Class XI) - Volume - III

1  GMm  GMm  GM 
E mv 02     as v 0  
2  r  2r  r 

(a) So, the energy required to place the lab from the surface of earth to the orbit of radius 2R,

GMm  GMm  3 GMm


E I  ES     
2  2R   R  4 R

3m 3  GM 
i.e., E   gR 2  mgR  as g  2 
4R 4  R 

i.e., E 
3
4
 2  103  10  6.4  106   12.8  1010   9.6  1010 J
3
4
(b) As for II orbit r < 3R
GMm GMm
E II   
2  3R  6R

GMm  GMm  1 GMm


 E II  E I    
6R  4R  12 R

 GM 
But as g   2  , i.e., GM  gR 2
 R 

mgR  12.8  1010   1.1 1010 J


1 1
or E 
12 12
Example 28: Two satellites of same mass are launched in the same orbit round the earth so as to rotate opposite to
each other. They soon collide inelastically and stick together as wreckage. Obtain the total energy of the system
before and just after the collision. Describe the subsequent motion of the wreckage.
Solution : In case of satellites of same mass are launched in the same orbit round the earth so as to rotate opposite
to each other. They soon collide inelastically and stick together as wreckage. Obtain the total energy of the system
before and just after the collision. Describe the subsequent motion of the wreckage.
Solution : In case of satellite motion energy of a satellite in an orbit is given by
GMm
E
2r
So the total energy of the system before collision.
GMm
E1  E1  E 2  2E  
r
As the satellites of equal mass are moving in opposite direction and collides inelastically, the velocity of
wreckage just after collision, by conservation of linear momentum will be
mv  mv  2mV , i.e., V = 0.

47
[J] Physics (Class XI) - Volume - III

i.e., just after collision wreckage comes to rest in the orbit. So energy of the wreckage just after collision will
be totally potential and will be
GM  2m  2GMm
EF   
r r
And as after collision the wreckage comes to stand still in the orbit, it will move along the radius towards the
earth under its gravity.
Weightlessness
The weight of a body is the force with which it is attracted towards the centre of earth. When a body is
stationary with respect to the earth, its weight equals the gravity. This weight of the body is known as its static or true
weight.
We become conscious of our weight, only when our weight (which is gravity) is opposed by some other object.
Actually, the secret of measuring the weight of a body with a weighing machine lies in the fact that as we place the
body on the machine, the weighing machine opposes the weight of the body. The reaction of the weighing machine
to the body gives the measure of the weight of the body.
The state of weightlessness can be observed in the following situations.
(1) When objects fall freely under gravity : For example, a lift falling freely, or an airship showing a feat in
which it falls freely for a few seconds during its flight, are in state of weightlessness.
(2) When a satellite revolves in its orbit around the earth : Weightlessness posses many serious problems to
the astronauts. It becomes quite difficult for them to control their movements. Everything in the satellite has to
be kept tied down. Creation of artificial gravity is the answer to this problem.
(3) When bodies are at null points in outer space : On a body projected up, the pull of the earth goes on
decreasing, but at the same time the gravitational pull of the moon on the body goes on increasing. At one
particular position, the two gravitational pulls may be equal and opposite and the net pull on the body becomes
zero. This is zero gravity region or the null point and the body in question is said to appear weightless.
Weightlessness in a Satellite.
A satellite, which does not produce its own gravity moves around the earth in a circular orbit under the action
GM
of gravity. The acceleration of satellite is towards the centre of earth.
r2 r

If a body of mass m placed on a surface inside a satellite moving around the earth. Then force on the body are
GMm
(i) The gravitational pull of earth 
r2
(ii) The reaction by the surface = R

48
[J] Physics (Class XI) - Volume - III

GmM
By Newtons law  R  ma
r2
GmM  GM 
 R  m 2   R = 0
 r 
2
r
Thus the surface does not exert any force on the body and hence its apparent weight is zero.
A body needs no support to stay at rest in the satellite and hence all position are equally comfortable. Such a state
is called weightlessness.
Important points
(i) One will find it difficult to control his movement, without weight, he will tend to float freely. To get from one spot
to the other he will have to push himself away from the walls or some other fixed objects.
(ii) As everything is in free fall, so objects are at rest relative to each other.
(iii) If a glass of water is tilted and glass is pulled out, the liquid in the shape of container will float and will not flow
because of surface tension.
(iv) If one tries to strike a match, the head will light but the stick will not burn. This is because in this situation
convection currents will not be set up which supply oxygen for combustion
(v) If one tries to perform simple pendulum experiment, the pendulum will not oscillate. It is because there will not
be any restoring torque and so T  2  L / g    [As g’ = 0 ]
(vi) Condition of weightlessness can be experienced only when the mass of satellite is negligible so that it does not
produce its own gravity.
e.g. Moon is a satellite of earth but due to its own weight it applies gravitational force of attraction on the body
placed on its surface and hence weight of the body will not be equal to zero at the surface of the moon.
Example 29 : An object weighs 10 N at the northpole of the earth. In a geostationary satellite distant 7R from the
centre of earth (of radius R) what will be its (a) true weight, (b) apparent weight?
Solution : (a) The true weight of a body is given by mg and with height ‘g’ decrease:

WS mg 1  g 
   as g  
So, W mg 1   h / R   2     
2
E
   
1  h / R 

h
But here h = 7R – R = 6 R, i.e., 6
R
WE 10
So, WS  1  6 2  49  0.2N
 

49
[J] Physics (Class XI) - Volume - III

QUESTIONS
LEVEL - I
1. ve and vp denote the escape velocities from the earth and another planet having twice the radius and the same
mean density as that of the earth. Then
vp vp
A) ve  B) v e  v p C) v e  2v p D) ve 
2 4
2. Time period of revolution of a satellite around a planet of radius R is T. Period of revolution around another
planet, whose radius is 3R but having same density is
A) T B) 3T C) 9T D) 3 3 . T
3. Potential energy of a satellite having mass ‘m’ and rotating at a height of 6.4 × 106 m from the earth surface is
A) –0.5 mgRe B) –mgRe C) –2mgRe D) 4mgRe
4. If earth is at one-fourth of its present distance from the sun, the duration of the year will be
A) One-half the present year B) One-fourth the present year
C) One-sixth the present year D) One-eighth the present year
5. Two satellites A and B go round a planet P in circular orbits having radii 4R and R respectively. If the speed of
the satellite A is 3v, the speed of satellite B will be
4v 3v
A) 12 v B) 6 v C) D)
3 2
6. The largest and the shortest distances of the earth from the sun are r1 and r2. Its distance from the sun when it
is at the perpendicular to the major axis of the orbit drawn from the sun

r1  r2 2 r1 r2 r1r2 r1  r2
A) B) r  r C) r  r D)
4 1 2 1 2 3
7. The orbital velocity of an artificial satellite in a circular orbit just above the earth’s surface is v. For a satellite
orbiting at an altitude of half of the earth’s radius, the orbital velocity is

3  3 2
A) v B) .v C) .v D) v
2 3 2 3
8. An earth satellite S has an orbit radius which is 4 times that of a communication satellite C. The period of
revolution of S is
A) 2 day B) 4 day C) 8 day D) 16 day
9. The radii of two planets are R1 and R2. Their densities are respectively 1 and 2 . The ratio of the acceleration
due to gravity at their surface is
g1 R 22 g1 R 21 g1 R 11 g1 R 12
A) g  R  B) g  R  C) g  R  D) g  R 
2 1 1 2 1 2 2 2 2 2 2 1

10. The period of revolution of planet A around the sun is 8 times that of B. The distance of A from the sun is
............ times greater than that of B from the sun?
A) 4 B) 5 C) 6 D) 8

50
[J] Physics (Class XI) - Volume - III

11. An artificial satellite revolving in a circular orbit around the earth has a total energy E0, being the sum of P.E. and
K.E. Its potential energy is
E0
A) 2 E 0 B) E 0 2 C)  D) 2 2 E 0
2
12. The orbital angular momentum of a satellite revolving at a distance r from the centre is L. If the distance is
increased to 16 r, then new angular momentum will be
L
A) 16 L B) 64 L C) D) 4 L
4
13. A mass M is split into two parts, m and (M – m). They are then separated by a certain distance. The gravitational
m
force between these two parts will be maximum if the ratio is equal to
M
1 1 1 1
A) B) C) D)
4 3 2 1
14. The earth revolves round the sun is one year. If the distance between them becomes double, the new period of
revolution will be
A) 8 year B) 4 year C) 2 2 year D) 2 year
15. If the radius of earth shrinks by 1.5% (mass remaining same), then the value of acceleration due to gravity
changes by
A) 1% B) 2% C) 3% D) 4%
16. Select the proper graph between the gravitational potential (Vg) due to hollow sphere and distance (r) from its
centre.

17. The escape velocity of a particle of mass m varies as


A) m B) mo C) m–1 D) m–2
18. In a satellite if the time of revolution is T, then kinetic energy is proportional to
A) T–1 B) T–2/3 C) T–2 D) T–1/3
19. Two identical satellites A and B are circulating round the earth at the height of R and 2R respectively (where R
is the radius of earth). The ratio of kinetic energy of A to that of B is
2 3 3 5
A) B) C) D)
3 2 5 3
20. A satellite is revolving round the earth in circular orbit at some height above surface of earth. It takes 5.26 × 103
seconds to complete a revolution while its centripetal acceleration is 9.32 m/s2. Height of satellite above
surface of earth is (Radius of earth 6.37 × 106 m)
A) 70 km B) 120 km C) 160 km D) 220 km

51
[J] Physics (Class XI) - Volume - III

21. The height of the point vertically above earth’s surface at which acceleration due to gravity becomes 1% of its
value at the surface is (Radius of earth = R)
A) 8R B) 9R C) 10 R D) 20 R
22. If mass and diameter of a planet are twice those of earth, the period of oscillation of pendulum on this planet will
be (If it is a second’s pendulum on earth)
1 1
A) sec B) 2 sec C) sec D) 2 2 sec
2 2
23. If radius of earth is R, then the height h at which value of g becomes one-fourth is
R R 3R
A) B) D) D) R
4 2 4
24. Two identical satellites A and B revolve round the earth in circular orbits, at distances R and 3R from the
surface of the earth, where R denotes radius of earth. The ratio of angular momenta of A and B is
A) 1 : 1 B) 1: 2 C) 2 :1 D) 2 : 1
25. ve denotes escape velocity for a planet. A tunnel is dug along a diameter of the planet. A small body is dropped
into it at the surface. The speed of body, when it reaches the centre of planet, will be
ve ve
A) 2ve B) 2 ve C) D)
2 2
26. An artificial satellite of mass m orbits earth at a height R above the surface of earth. If g denotes gravitational
field intensity at the surface of earth of radius R, the kinetic energy of the revolving satellite will be
mgR mgR mgR mgR
A) B) C) D)
4 2 3 5
27. A geostationary satellite orbits earth at a height of 6R above the surface of earth where R denotes radius of
earth. Another satellite revolves around earth at a height 2.5R from earth’s surface. The period of other satellite
will be
A) (6 × 2.5) hour B) (6/2.5) hour C) (2.5/6) hour D) 6 2 hour
28. The magnitudes of gravitational fields at distances r1 and r2 from the centre of a uniform sphere of radius R are
F1 and F2. The ratio F1/F2 when r1 < R and r2 < r1, is
2
r1 r1  r1  r2
A) r B) C)   D)
2 r2  r2  r1
29. g is the acceleration due to gravity at a point very near earth’s surface. The earth suddenly shrinks to a smaller
radius (x) where (x) = 0.8 R, where R denotes radius of earth, without change in its mass. Now, the acceleration
due to gravity at the same point will be
A) 0.8 g B) 0.4 g C) g D) g/2
30. Two particles of equal mass m each, go round a circle of radius R, under the action of their mutual gravitational
attraction. The speed of each particle is

Gm 1  1 Gm 2Gm
A) B) C) D)
2R 2R Gm 2 R R
52
[J] Physics (Class XI) - Volume - III

31. The correct graph representing the variation of total energy (E), kinetic energy (K) and potential energy (U) of
a satellite with its distance from the centre of earth is

32. A body of mass m is taken from earth surface to a height equal to radius of earth. The increase in potential
energy will be
mgR mgR
A) mgR B) C) 2mgR D)
2 4
33. The change in potential energy when a body of mass m is raised to a height (nR) from the earth’s surface is
(radius of earth = R)

mgR  n  1 mgR  n  1 mgRn mgRn


A) B) C)  n  1 D)  n  1
n n
34. There are two bodies of masses 100 kg and 10,000 kg separated by a distance of 1 m. At what distance from
the smaller body, the intensity of gravitational field will be zero?
1 1 1 10
A) m B) m C) m D) m
9 10 11 11
35. The metallic bob of simple pendulum has the relative density  . The time period of this pendulum is T. If the
metallic bob is immersed in water, then the new time period will be

T    1 T    1 
C) T D) T
A)

B)
(  1)     1
36. A satellite is placed in a circular orbit around earth at such a height that it always remains stationary with respect
to earth’s surface. Its height from earth’s surface is
A) 32,000 km B) 36,000 km C) 6400 km D) 42400 km
37. Geostationary satellite orbits around the earth in a circular orbit of radius 42400 km. Then the time period of a
spy satellite orbiting a few hundred km above earth’s surface (R = 6400 km) will approximately be
A) 1/2 h B) 3 h C) 2 h D) 4 h
38. A pendulum is taken inside 1 km from sea level. Then, in 1 day, it
A) Loses 13.5 sec B) Gains 13.5 sec C) Loses 7 sec D) Gains 7 sec.
39. Two spheres, each of radius r, are touching each other. The force of attraction between them is proportional to
A) r6 B) r4 C) r2 D) r–2
40. Energy required to move a body of mass m from an orbit of radius 2R to 3R is (where M = Mass of the earth,
R = Radius of the earth)
GMm GMm GMm GMm
A) B) C) D)
12R 3R 8R 6R
53
[J] Physics (Class XI) - Volume - III

41. Infinite number of masses, each of 3 kg, are placed along a straight line at the distances of 1 m, 2m, 4 m, 8m,
.... from a point O on the same line. If G denotes the universal constant of gravitation, then the magnitude of the
gravitational field intensity at O is
A) G B) 2 G C) 3 G D) 4 G
42. Infinite number of masses, each of 1 kg, are placed along a straight line at the distances of 1 m, 2 m, 4 m, 8
m,..... from a point O on the same time. If G denotes universal gravitational constant, then the gravitational
potential at O is
A) – G B) –2G C) –3G D) –4G
43. A satellite goes along an elliptic path around earth. The rate of change of area swept by the line joining earth and
the satellite is proportional to
A) r B) r C) r2 D) r3
44. A small body of super dense material has a mass twice the mass of earth. Its size is very small as compared to
the size of earth. While at a height H << R above the earth’s surface, it starts from rest and reaches the earth’s
surface in time given by

H H 2H 4H
A) B) C) D)
g g 3g 3g
45. Mass M is divided into two parts xM and (1 – x)M. For a given separation, the value of x for which the
gravitational attraction between the two pieces becomes maximum is
1 3
A) B) C) 1 D) 2
2 5
46. Three identical point masses, each of mass 1 kg lie in the x-y plane at points (0, 0), (0, 0.2 m) and (0.2 m, 0).
The net gravitational force on the mass at the origin is

A) 1.67 10
9
 ˆi  ˆj N B) 3.34 10
10 ˆ
 
i  ˆj N

C) 1.67  10
9
 ˆi  ˆj N D) 3.34 10
10
 ˆi  ˆj N
47. Four particles of mass m, 2m, 3m and 4m are kept in sequence at the corners of a square of side a. The
magnitude of gravitational force acting on a particle of mass m placed at the centre of the square will be

24m 2 G 6m 2 G 4 2 Gm 2
A) B) C) D) Zero
a2 a2 a2
48. Acceleration due to gravity on moon is 1/6 of the acceleration due to gravity on earth. If the ratio of densities of
 e  5
earth  m  and the moon  e  is    then radius of moon Rm in terms of Re will be
 m  3

5 1 3 1
A) Re B) Re C) Re D) Re
18 6 18 2 3
49. A spherical planet far out in space has a mass M0 and diameter D0. A particle of mass m falling freely near the
surface of this planet will experience an acceleration due to gravity which is equal to
A) GM 0 / D 02 B) 4mGM 0 / D 02 C) 4GM 0 / D 02 D) GmM 0 / D 02
54
[J] Physics (Class XI) - Volume - III

50. The moon’s radius is 1/4 that of the earth and its mass is 1/80 times that of the earth. If g represents the
acceleration due to gravity on the surface of the earth, that on the surface of the moon is
g g g g
A) B) C) D)
4 5 6 8

LEVEL - II
1. Two particles of equal mass go round a circle of radius R under the action their mutual gravitational attraction.
The speed of each particle is

1  Gm 1 Gm 4Gm
A) v  B) v  C) v  D) v 
2R Gm 2R 2 R R
2. Mass of moon is 7.34 × 1022 kg. If the acceleration due to gravity on the moon is 1.4 m/s2, the radius of the
moon is (G = 6.667 × 10–11 Nm2/kg2)
A) 0.56 × 104 m B) 1.87 × 106m C) 1.92 × 106m D) 1.01 × 108 m
3. A planet has mass 1/10 of that of earth, while radius is 1/3 that of earth. If a person can throw a stone on earth
surface to a height of 90 m, then he will be able to throw the stone on that planet to a height
A) 90 m B) 40 m C) 100 m D) 45 m
4. The radii of two planets are respectively R1 and R2 and their densities are respectively 1 and 2 . The ratio of
the acceleration due to gravity at their surface is
1 2
A) g1 : g 2  R 2 : R 2 B) g1 : g 2  R1R 2 : 12
1 2

C) g1 : g 2  R12 : R 21 D) g1 : g 2  R11 : R 22


5. The acceleration of a body due to the attraction of the earth (radius R) at a distance 2R from the surface of the
earth is (g = acceleration due to gravity at the surface of the earth)
g g g
A) B) C) D) g
9 3 4
R
6. At surface of earth weight of a person is 72 N then his weight at height from surface of earth is (R = radius
2
of earth)
A) 28 N B) 16 N C) 32 N D) 72 N
7. If the distance between centres of earth and moon is D and the mass of earth is 81 times the mass of moon, then
at what distance from centre of earth the gravitational force will be zero
D 2D 4D 9D
A) B) C) D)
2 3 3 10
8. Weight of a body of mass m decreases by 1% when it is raised to height h above the earth’s surface. If the body
is taken to a depth h in a mine, change in its weight is
A) 2% decrease B) 0.5% decrease C) 1% increase D) 0.5% increase

55
[J] Physics (Class XI) - Volume - III

g
9. The depth at which the effective value of acceleration due to gravity is is (R = radius of the earth)
4

3R R R
A) R B) C) D)
4 2 4
10. Assuming earth to be a sphere of uniform density, what is the value of gravitational acceleration in a mine 100
km below the earth’s surface (Given R = 6400 km)

A) 9.66 m / s 2 B) 7.64 m / s 2 C) 5.06 m / s 2 D) 3.10 m / s 2

1
11. The depth d at which the value of acceleration due to gravity becomes times the value at the surface,
n
is [R = radius of the earth]

R  n 1  R  n 
A) B) R   C) D) R  
n  n  n2  n 1
12. The angular velocity of the earth with which it has to rotate so that acceleration due to gravity on 60o lattitude
becomes zero is (Radius of earth = 6400 km. At the poles g = 10 ms–2)
A) 2.5 × 10–3 rad/sec B) 5.0 × 10–3 rad/sec C) 10 × 101 rad/sec D) 7.8 × 10–2 rad/sec

M
13. Knowing that mass of Moon is where M is the mass of Earth, find the distance at the point where gravitational
81
field due to Earth and Moon cancel each other, from the Moon. Given that distance between Earth and Moon
is 60 R. Where R is the radius of Earth.
A) 2 R B) 4 R C) 6 R D) 8 R
14. The gravitational potential in a region is given by V = (3x + 4y + 12z).J/kg. The modulus of the gravitational
field at (x = 1, y = 0, z = 3) is
A) 20 N kg–1 B) 13 N kg–1 C) 12 N kg–1 D) 5 N kg–1
15. A s p a c e s h i p i s o r b i t i n g t h e e

c
= 6400 km) from
a r t h i n a c i r c u l a r o r b i t a t a h e i g h t e q u a l t o r a d i u s o f t h e e a r t h ( R

the surface of the earth. An astronaut is on a space walk outside the spaceship. He is at a distance of l = 200m
from the ship and is connected to it with a simple cable which can sustain a maximum tension of 10 N. Assume
that the centre of the earth, the spaceship and the astronaut are in a line. Mass of astronaut along with all his
accessories is 100 kg. Tension in the cable is :
A) 0.01 N B) 0.001 N C) 0.1 N D) 1 N

1
16. If the radius of earth contracts of its present value, the length of the day will be approximately
n

24 24
A) h B) h C) 24 nh D) 24n2 h
n n2

56
[J] Physics (Class XI) - Volume - III

17. Two concentric shells of mass M1 and M2 are having radii r1 and r2. Which of the following is the correct
expression for the gravitational field on a mass m.

G  M1  M 2  G  M1  M 2 
A) I  for r  r1 B) I  for r  r2
r2 r2

M2 GM1
C) I  G for r1  r  r2 D) I  for r1  r  r2
r2 r2
18. A spherical shell is cut into two pieces along a chord as shown in the figure. P is a point on the plane of the
chord. The graviational field at P due to the upper part is I1 and that due to the lower part is I2. What is the
relation between them

A) I1 > I2 B) I1 < I2 C) I1 = I2 D) No definite relation


19. A uniform ring of mass m is lying at a distance 1.73 a from the centre of a sphere of mass M just over the sphere
where a is the small radius of the ring as well as that of the sphere. Then gravitational force exerted on the
sphere is

GMm GMm GMm GMm


3 D) 1.73
A)
8a 2  
B) 1.73a 2 C)
a2 8a 2
20. In some region, the gravitational field is zero. The gravitational potential in this region.
A) Must be variable B) Must be constant C) Cannot be zero D) Must be zero
K
21. The gravitational field due to a mass distribution is E  in the x-direction (K is a constant), Taking the
x3
gravitational potential to be zero at infinity, its value at a distance x is
K K K K
A) B) C) 2 D) 2x 2
x 2x x
57
[J] Physics (Class XI) - Volume - III

22. The intensity of gravitational field at a point situated at a distance of 8000 km from the centre of the earth is 6N/
kg. The gravitational potential at that point is – (in Joule / kg)
A) 8 × 106 B) 2.4 × 103 C) 4.8 × 107 D) 6.4 × 1014
23. A body released from a height h takes time t to reach earth’s surface. The time taken by the same body released
from the same height to reach the moon’s surface is
t
A) t B) 6t C) 6t D)
6
24. Two bodies of mass m and M are placed at distance d apart. The gravitational potential at the position where
the gravitational field due to them is zero is V, then

G
m  M
Gm GM G
  M
2
A) V   B) V   C) V   D) V   m M
d d d d m
25. A satellite is revolving round the earth with orbital speed v0. If it stops suddenly, the speed with which it will
strike the surface of earth would be (ve = escape velocity of a particle on earth’s surface)

v e2
A)
v0
B) v 0 C) ve2  v02 D) ve2  2v02

26. A body of mass m kg. starts falling from a point 2R above the earth’s surface. Its kinetic energy when it has
fallen to a point ‘R’ above the earth’s surface [R-Radius of earth, M-Mass of earth, G-Gravitational constant]
1 GMm 1 GMm 2 GMm 1 GMm
A) B) C) D)
2 R 6 R 3 R 3 R
27. A body of mass m is taken from earth surface to the height h equal to radius of earth, the increase in potential
energy will be
1 1
A) mgR B) mgR C) 2 mgR D) mgR
2 4
28. If mass of earth is M, radius is R and gravitational constant is G, then work done to take 1 kg mass from earth
surface to infinity will be

GM GM 2GM GM
A) B) C) D)
2R R R 2R
29. Three paticles each of mass 100 gm are brought from a very large distance to the vertices of an equilateral
triangle whose side is 20 cm in length. The work done will be
A) 0.33 × 10–11 J B) –0.33 × 10–11 J C) 1.00 × 10–11 J D) –1.00 × 10–11 J
30. A boy can jump to a height h on ground level. What should be the radius of a sphere of density d such that on
jumping on it, he escapes out of the gravitational field of the sphere
1/2 1/2
 4 Gd   4 gh   3 Gd 
1/2 1/2
 3 gh 
A)   B)  D)  
 3 Gd 
C) 
 3 gh   4 Gd   4 gh 
58
[J] Physics (Class XI) - Volume - III

31. The escape velocity from the earth is about 11 km/s. The escape velocity from a planet having twice the radius
and the same mean density as the earth, is
A) 22 km/s B) 11 km/s C) 5.5 km/s D) 15.5 km/s
32. A projectile is projected with velocity kve in vertically upward direction from the ground into the space. (ve is
escape velocity and k < 1). If air resistance is considered to be negligible then the maximum height from the
surface of the earth to which it can go, will be (R = radius of earth)

R R Rk 2 R
A) 2
k 1
B) 2
k 1  
C) 1  k 2 D)
k 1
33. A rocket of mass M is launched vertically from the surface of the earth with an initial speed V. Assuming the
radius of the earth to be R and negligible air resistance, the maximum height attained by the rocket above the
surface of the earth is
R R
 gR   2gR 
A)  gR  1 B) R   1 C)  2gR  1 D) R  2  1
 2V   V 
2
 2V   V 
2 2

34. A body of mass m is situated at a distance 4Re above the earth’s surface, where Re is the radius of earth. How
much minimum energy be given to the body so that it may escape.
mgR e mgR e
A) mgRe B) 2mgRe C) D)
5 16
35. The distance of a planet from the sun is 5 times the distance between the earth and the sun. The time period of
the planet is
A) 53/2 years B) 52/3 years C) 51/3 years D) 51/2 years
36. In planetary motion the areal velocity of position vector of a planet depends on angular velocity   and the
distance of the planet from sun (r). If so the correct relation for areal velocity is
dA dA dA dA
A)  r B)  2 r C)  r 2 D)  r
dt dt dt dt
37. The distance of Neptune and Saturn from sun are nearly 1013 and 1012 meters respectively. Assuming that they
move in circular orbits, their periodic times will be in the ratio
A) 10 B) 100 C) 10 10 D) 1/ 10
38. The maximum and minimum distance of a comet from the sun is 8 × 1012 m and 1.6 × 1012 m. If its velocity
when nearest to the sun is 60 m/s, what will be its velocity in m/s when it is farthest
A) 12 B) 60 C) 112 D) 6
39. A satellite A of mass m is at a distance of r from the centre of the earth. Another satellite B of mass 2 m is at
distance of 2r from the earth’s cente. Their time periods are in the ratio of
A) 1 : 2 B) 1 : 16 C) 1 : 32 D) 1: 2 2
40. A planet moves around the sun. At a given point P, it is closed to the sun at a distance d1 and has a speed v1. At
another point Q, when it is farthest from the sun at a distance d2, its speed will be
d12 v1 d 2 v1 d1v1 d 22 v1
A) 2 B) d C) d D) 2
d2 1 2 d1
59
[J] Physics (Class XI) - Volume - III

41. Two satellites A and B go round a planet P in circular orbits having radii 4 R and R respectively. If the speed of
the satellite A is 3 v, the speed of the satellite B will be
A) 12 v B) 6 v C) 3 v D) 2 v
42. If the gravitational force between two objects were proportional to 1/R; where R is separation between them,
then a particle in circular orbit under such a force would have its orbital speed  proportional to

1 1
A) B) Ro C) R1 D)
R2 R

43. The distance between centre of the earth and moon is 384000 km. If the mass of the earth is 6 1024 kg and
G  6.67 1011 Nm 2 / kg 2 . The speed of the moon is nearly
A) 1 km/sec B) 4 km/sec C) 8 km/sec D) 11.2 km/sec
44. A satellite is launched into a circular obrit of radius ‘R’ around earth while a second satellite is launched into an
orbit of radius 1.02 R. The percentage difference in the time periods of the two satellite is
A) 0.7 B) 1.0 C) 1.5 D) 3
45. Periodic time of a satellite revolving above Earth’s surface at a height equal to R, where R the radius of Earth,
is [g is acceleration due to gravity at Earth’s surface]

2R R R R
A) 2 B) 4 2  C) 2 D) 8
g g g g

46. An earth satellite S has an orbit radius which is 4 times that of a communication satellite C. The period of
revolution of S is
A) 4 days B) 8 days C) 16 days D) 32 days
47. One projectile after deviation from its path, starts moving round the earth in a circular path at radius equal to
nine times the radius at earth R, its time period will be

R R R R
A) 2 B) 27  2 C)  D) 8  2
g g g g

48. The gravitational potential difference between the surface of a planet and a point 20 m above it is 14 J kg–1.
The work done in moving a 2.0 kg mass by 8.0 m on a slope of 60o from the horizontal, is equal to
A) 7 J B) 9.6 J C) 16 J D) 32 J
49. Given radius of earth ‘R’ and length of a day ‘T’ the height of a geostationary satellite is
[G - Gravitational constant, M - Mass of earth]
1/3 1/3 1/3
 42 GM   4GM 
1/3
 GMT 2   GMT 2 
A)   B)   R C)   R D)   R
 4   4 
2 2 2
 T   R 
2

50. A satellite is moving around the earth with speed  in a circular orbit of radius r. If the orbit radius is decreased
by 1%, its speed will
A) Increase by 1% B) Increase by 0.5% C) Decrease by 1% D) Decrease by 0.5%

60
[J] Physics (Class XI) - Volume - III

LEVEL III
Single correct option.
1. A b i n a r y s tm and nm (where n is a numerical factor) having seperation of their centres as
a r h a s s t a r s o f m a s s e s

r . If these stars revolve because of gravitational force of each other, the period of revolution is given by
3 1
2r 2
2r 2 3
2r 3 2r 2
A) 1 B) 1 C) D)
 Gnm 2   Gm 1  n  
1
2 2
3 2
2
  Gnm
 
 m 1  n  
2 3  GMn 
 nm  2 
2. Two blcoks of masses m each are hung from a balance. The scale pan A is at height H1 whereas scale pan B is
at height H2. The difference in weight when H1 > H2 and R being the radius of earth =

 1  2H1   H1 H 2   H 2 H1  H 2 H1
A) mg   B) 2mg    C) 2mg    D) 2mg H  H
 R  R R   R R  1 2

3. Velocity of a body released from a distance d measured from the centre of earth of mass M and radius R when
it strikes the surface of earth is given by  is

1 1   1 1  1 1 1 1 
A) GM    2GM    C) 3GM    D) 2GM   
B)
d R R d R d d R
4. Work done to bring 4 particles each having a mass of 0.1 kg from infinity to the vertices of sqaure of side 0.2
m is

A) 1.8  1011 J B) 8.1 1011 J C) 1.8  10 11 J D) 8.1 1011 J


61
[J] Physics (Class XI) - Volume - III

5. T w o p l a n e t s A a n d B a r e a

A
and that of B is MB, then the
t a d i s t a n c e d f r o m e a c h o t h e r . I f t h e m a s s o f A i s M

escape velocity of a satellite of mass MS projected from the mid point of two planets is

2G  M A  M B  G  MA  MB  G  MA  MB  2GM A M B
A)
d
B) 2
d
C)
d
D)
 MA  MB  d
6. A projectile is fired from the surface of earth of radius R with a velocity ke where e is the escape velocity
and k < 1. Neglecting air resistance, the maximum height of rise from centre of earth is
R R
A) B) k2R C) D) kR
k 1 2
1 k2
7. A mass m is at a distance a from one end of a uniform rod of length l and mass M. The gravitational force on the
mass due to rod is
M a

m
l
GMm
GMm GmM GMm A)
A) l 
a  l  B) a  l  a  C)
a
D)  a
2 
8. A metal sphere has radius R and mass M. A spherical hollow of diameter R is made in this sphere such that its
surface passes through the centre of the metal sphere and touches the outside surface of the metal sphere. A
unit mass is placed at a distance a from the centre of metal sphere. The gravitation field at that point is

Unit mass
R R

   
   
GM  1  GM  1 
1  1 
A) R 2
  2R  
2
B) a 2
  2R  
2

 8 1     8 1   
  a     a  
   
   
GM  1  GM  1 
1 1
B)  R  a 2   R 
2
D)  a  R 2   2a  
2

 8 1     8 1   
  2a     R 

62
[J] Physics (Class XI) - Volume - III

9. Four similar particles of mass m are orbiting in a circle of radius r in the same direction because of their mutual
gravitational attractive force. Velocity of a particle is given by

1 1
 Gm  1  2 2   2  1 GM  1  2   2
A)     B) 3
Gm
C)
Gm

1 2 2  D)    
 r  4   r r  2 r  2  
10. The pressure caused by gravitational pull inside earth at a distance a measured from its centre, when its mass,
density and radius are M,  and R respectively, is given by

8 GM 2  a 2  2 a2  3 GM 2   a  
2
3 8 GM 2  a
A) 1   B) GM  1  2  C) 8 R 4 1   R   D) 3 
1 
3 R 3  R 2  8  R      3 R  R

11. The largest and shortest distance of the earth from sun are a and b respectively. Then distance of earth from sun
when it is at a point where its distance from major axis is same as that of perpendicular distance of sun from the
major axis

1 a  b
2
ab 1 ab 2ab
A) B) C) D)
ab 2 ab ab 2 ab
12. The percentage change in the acceleration of the earth towards the sun from a total eclipse of the sun to the
point where the moon is on a side of earth direclty opposite to the sun is

2 2 2
M s r2  r2  M s  r1  M m  r1  M m
A) M r  100 B)    100 C) 2    100 D)    100
m 1  r1  M m  r2  M s  r2  M s
13. The gravitational potential of two homogeneous spherical shells of same surface density at their respective
centres are in the ratio 3 : 4. If the two shells coalesce into single one, the potential of an internal point of the
shell when surface density remains unchanged is
A) 1 : 2 : 3 B) 2 : 3 : 4 C) 3 : 4 : 5 D) 4 : 5 : 6

63
[J] Physics (Class XI) - Volume - III

14. A straight rod of length l extends from x   to x  l   . If the mass per unit length is (a + bx2). The
gravitational force it exerts on a point mass m placed at x = 0 is given by

 1 1   Gm  a  bx 2 
A) Gm  a     bl  B)
  l   l2

 1 1     1 1 
C) Gm       bl  D) Gm  a     bl 
 a a l    l   
15. Two masses of m each are placed at a separation distance of 2d. A small mass ms placed midway, when
displaced, starts oscillating. Then

1 2Gm
A) Frequency of non simple harmonic motion is given by
2 d 3

1 2Gm
B) Frequency of simple harmonic motion is given by
2 d 3

Gm
C) Acceleration of the mass ms is given by
d2

GM
D) Time period of vibration is 2
 2d 
3

16. A uniform sphere of mass M and radius R exerts a force F on a small mass m situated at a distance of 2R from
the centre O of the sphere. A spherical portion of diameter R is cut from the sphere as shown in Fig. The force
of attraction between the remaining part of the sphere and the mass m will be

7F 2F 4F F
A) B) C) D)
9 3 9 3

64
[J] Physics (Class XI) - Volume - III

17. Imagine a light planet revolving around a very massive star in a circular orbit of radius R with a period of
revolution T. If the gravitational force of attraction between the planet and the star is proportional to R–5/2 then
A) T2 is proportional to R3 B) T2 is proportional to R7/2
C) T2 is proportional to R3/2 D) T2 is proportional to R3/75
18. Two satellites are moving around the earth in circular orbits at height R and 3R respectively, R being the radius
of the earth, the ratio of their kinetic energies is
A) 2 B) 4 C) 8 D) 16
19. A small body of superdense material, whose mass is twice the mass of the earth but whose size is very small
compared to the size of the earth, starts from rest at a height H < < R above the earth’s sruface. At what time
it will reach the surface of the earth.

2H H 2H 3H
A) B) C) D)
g g 3g 2g
20. A s i m p l e p e n d when on earth’s surface and T2 when taken to a height R above the
u l u m h a s a t i m e p e r i o d T

1
earth’s surface, where R is the radius of earth. The value of T2/T1 is
A) 1 B) 2 C) 4 D) 2
More than one correct option.
21. The magnitudes of the gravitational field at distances r1 and r2 from the centre of a uniform sphere of radius R
and mass M are F1 and F2 respectively; then:

 F1   r1 
2
 F1   r2 
A)      if r1  R and r2  R B)      if r1  R and r2  R
 F2   r2   F2   r1 

 F1   r1 
2
 F1   r1 
C)      if r1  R and r2  R D)      if r1  R and r2  R
 F2   r2   F2   r2 
22. Which of the following statements are true about acceleration due to gravity?
A) ‘g’ decreases in moving away from the centre if r > R
B) ‘g’ decreases in moving away from the centre of r < R
C) ‘g’ is zero at the centre of earth.
D) ‘g’ decreases if earth stops rotating on its axis
23. Which of the following statements are correct about a planet rotating around the sun in an elliptic orbit?
A) Its mechanical energy is constant B) Its angular momentum is constant
C) its areal velocity is constant D) Its time period is proportional to r3.
65
[J] Physics (Class XI) - Volume - III

24. In case of an orbiting statellite if the radius of orbit is decreased:


A) its KE decreaes B) its PE decreases C) its ME decreases D) its speed increases
25. Consider an attractive force which is central but is inversely proportional to the first power of distance. If such
a particle is in circular orbit under such a force, which of the following statements are correct?
A) The speed is directly proportional to the square root of orbital radius
B) The speed is independent of radius
C) The period is independent of radius
D) The period is directly proportional to radius
26. A tunnel is dug along a chord of the earth at a perpendciular distance R/2 from the earth’s centre. The wall of
the tunnel may be assumed to be frictionless. A particle is released from one end of the tunnel. The pressing
force by the particle on the wall and the acceleration of the particle varies with x (distance of the particle from
the centre of the earth) according to :

Passage Comprehension Type


Passage 1
In a planetary and stellar astronomy, a ring of matter is a very common and familiar system, eg., Saturn’s rings.
Consider a uniform ring of mass M and radius R. A particle of mass ‘m’ is released at rest at a point on the axis
of ring at a distance ‘x’ from the centre, as in Fig.

Answer these questions.


27. Force acting on the particle:
GMm
A) has a magnitude and always acts towards the centre of ring
x  R2 
2 3/ 2

GMm
x  R2 
B) has a magnitude 2 1/2 and acts towards centre when the particle is to the right of ring but acts away

from centre when the particle is to the left of ring.


GMmx
C) has a magnitude and always acts towards the centre
x  R2 
2 3/ 2

GMmx
x  R2 
D) has a magnitude 2 1/2 and always acts towards the centre

66
[J] Physics (Class XI) - Volume - III

28. U being potential energy of the system , the particle is in equilibrium

d2U
A) at x = 0 and  0 at this equilibrium
dx 2

d2U
B) at x = 0 and  0 at this equilibirum
dx 2

d2U
C) at x = 0 and  0 at this equilibrium
dx 2

d2U
D) at x   R and  0 at this equilibrium
dx 2
Passage 2
A satellite is revolving in an elliptical orbit around the earth, earth being at any focus of the ellipse. At perihelion,
which is that point in the satellite’s orbit at which it is closest to the earth, it is 1200 km above the surface of
earth. At aphelion, a point in the satellite’s orbit at which it is most distant from the earth, it is 6000 km above
the earth’s surface. Given GME = 4 × 1014 N-m2/kg
(G is the universal gravitational constant and ME is the mass of earth.)
Radius of earth = 6400 km
29. Ratio of the speed of the satellite at perihelion to its speed at aphelion is:
A) 1.92 B) 1.45 C) 1.77 D) 1.63
30. Speed of the satellite at aphelion is nearly:
A) 4951 m/s B) 2951 m/s C) 1951 m/s D) 3951 m/s
Passage 3
Consider a binary system of stars X of mass MX and Y of mass MY. Their masses are different and they revolve
about their centre of mass. Separation between the stars is R. The distance of X from the centre of mass is four
times the distance of star Y from the centre of mass. Again assuming the dimensions of the stars to be much
smaller than their separation.
31. Orbital time period of star X can be expressed as:

4R 3/2 4R 3/2 2R 3/2 2R 3/2


A) 5GM B) 5GM C) GM D) 3GM
Y X X Y

32. Orbital time period of star Y can be expressed as:

2R 3/2 4R 3/2 2R 3/2 4R 3/2


A) B) C) D)
5GM Y GM Y 5GM X 5GM X

67
[J] Physics (Class XI) - Volume - III

Matrix Match Type.


33. Let V and E denote the gravittional potential and gravitational field respectively at a point due to certain
uniform mass distribution described in four different sitautions of column-I. Assume the gravitational potential at
infinity to be zero. The value of E and V are given in column-II. Match the following.
Column – I (Cause) Column – II (Effect)
A) At centre of thin spherical shell p) E=0
B) At centre of solid sphere q) E0
C) A solid sphere has a non-concentric r) V0
spherical cavity. At the centre of the
spherical cavity
D) At centre of line joining two point s) V =0
masses of equal magnitude

34. A satellite of mass m is orbiting the earth at a height h from its surface. (M is mass of the earth and its
radius is R)

Column – I Column – II
A) Kinetic energy of the satellite p) GMm

R  h
B) Potential energy of the satellite q) 1 GMm
2 R  h
C) Total energy of the satellite r) GMm
2R  h
D) The energy must be spent to pull the s) 2GMm
satellite out of the earth’s R  h
gravitational field

35. A particle is taken to a distance r (>R) from centre of the earth and R is the radius of the earth. It is given
velocity V which is perpendicular r . With the given values of V in column-I the resultant path of particle in
column-II. Here ‘G’ is the universal gravitational constant and ‘M’ is the mass of the earth.

Column – I Column – II
A) V  GM / r p) Elliptical
B) V  2GM / r q) Parabolic
C) V  2GM / r r) Hyperbolic
D) GM / r  V  2GM / r s) Circular

68
[J] Physics (Class XI) - Volume - III

36. Match the following

Where Q(r) is magnitude of physical quantity as a function of r (distance from of spherical distribution of
radius R).
37 Match the columns I and II.

Column – I Column – II
A) Elliptical orbit of planet p) Kinetic energy conservation
B) Circular orbit of satellite q) Angular momentum conservation
C) Escape velocity r) Independent of mass of particle /
satellite
D) Orbital velocity s) GM
R
t) Area velocity constant

69
[J] Physics (Class XI) - Volume - III

38. Match the columns I and II.

Column – I Column – II
A) Kinetic energy of a body p) must be zero
projected from surface of earth,
at large distance from surface of
earth
B) Gravitational potential energy of q) may be zero
a bound system
C) Change in potential energy of a r) Positive
point mass if left free to itself,
with time
D) Change in areal velocity of earth s) May be negative
as earth moves from apogee
towards perigee
t) Must be negative

Integer Answer type.

39. If a planet was suddenly stopped in its orbit supposed to be circular. It would fall onto the sun in a time

 
2 / n times the period of the planets revolution. The value of n is.

40. A mass of 6 × 1024 kg (= mass of earth) is to be compressed in a sphere in such a way that the escape velocity
from its surface is 3 × 108 m/s (equal to that of light). The radius of the sphere in mm is

41. A particle is fired vertically upwards from the surface of earth and reaches a height 6400 km. If R = 6400 km
and g at the surface of earth is 10 m/s2. Then the initial velocity of the particle in km/s is nearly equal to

42. An artificial satellite is moving in a circular orbit around the earth with a speed equal to half the magnitude of
escape velocity from the earth. If the satellite is stopped suddenly in its orbit and allowed to fall freely onto the
earth. The speed with which it hits the surface of the earth in km/s is (g = 9.8 ms–2 and RE = 6400 km)

6
43. Gravitational acceleration on the surface of a planet is g , where g is the gravitational acceleration on the
11
2
surface of the earth. The average mass density of the planet is times that of the earth. If the escape speed on
3
the surface of the earth is taken to be 11 kms–1, the escape speed on the surface of the planet in kms–1 will be:

44. A binary star consists of two stars A (mass = 2.2 Ms) and B (mass = 11 Ms), where Ms is mass of the sun. They
are separated by distance d and are rotating about their centre of mass which is stationary. The ratio of the total
angular momentum of the binary star to the angular momentum of star B about the cenre of the mass is:

70
[J] Physics (Class XI) - Volume - III

Reasoning Type.
A) Statement I is True, Statement II is True; Statement II is a correct explanation for Statement I
B) Statement I is True, Statement II is True; Statement II is not a correct explanation for Statement I
C) Statement I is True, Statement II is False.
D) Statement I is False, Statement II is True.
45. Statement I : Consider a planet in an elliptic orbit around sun. Its kinetic energy changes with time and
angular momentum remains constant.
Statement II : Gravitational force is a central force. As a result, no torque acts on the planet about the
position of sun and so angular momentum remains conserved.
46. Statement I : Geostationary satellites may be set up in equatorial plane in orbits of any radius more than
earth’s radius.
Statement II : Geostationary satellites have period of revolution of 24 hrs.
47. Statement I : Smaller the orbit of the planet around the sun, shorter is the time it takes to complete one
revolution.
Statement II : According to Kepler’s third law of planetary motion, square of time period is proportional to
cube of mean distance from sun.
48. Statement I : If earth suddenly stops rotating about its axis, then the value of acceleration due to gravity will
become same at all the places.
Statement II : The value of acceleration due to gravity is independent of rotation of earth.
49. Statement I : The time period of revolution of a satellite close to surface of earth is smaller than that revolving
away from surface of earth.
Statement II : The square of time period of revolution of a satellite is directly proportional to cube of its
orbital radius.
50. Statement I : The square of the period of revolution of a planet is proportional to the cube of its mean
distance from sun.
Statement II : The intenstiy of the gravitational field of the sun acting on the planet is proportional to the mass
of the sun and inverersely proportional to square of its distance from the sun; it also acts
towards the sun.

71
[J] Physics (Class XI) - Volume - III

KEY WITH HINTS


LEVEL - I

2GM 2G  4 3  8G
1. A Escape velocity ve     R    .R
R R 3  3
32G ve R 1
Similarly, v p  . R; Then, v  2R  2
3 p

GmM
2. A Centripetal force mR2 = gravitational force 
R2

GM 4 R 3 . 4 G 2 4G
2  3
 G  3
 G.  ;  or 
R 3 R 3 3 T 3
T does not depend on R. Hence T remains as such
GMm GMm GMm
3. A Potential energy   r

Re  h

2R e

gR e2 m
 Potential energy    0.5 mg R e
2R e

2 3
 T1   r1   1 
3 2
1 1 T1 1
4. D By Kepler’s law, T 2  r 3 ;             
 T2   r2   4  64  8  T2 8

GM v1 R2 3v R 1
5. B Orbital velocity   v;      v  6v
R v2 R1 v2 4R 2 2

6. B The earth revolves around the sun in elliptical orbit. By using the property of eccentricity of ellipse, we
know
r1  r2
r1  1  e  a, r2  1  e  a  a  , r1r2  1  e 2  a 2
2
where a = semi major axis, b = semi minor axis, e = eccentricity

a 2 1  e 2  r1r2 2r r
b 2
d   12
Required distance = Semilatus rectum  ; a r1  r2 r1  r2
a 2

GM GM v0 2 2
7. B v0  v0     v0  . v0
R 3R / 2 v0 3 3
2
 TS 
Apply Kepler’s third law,     4   64   8 
3 2
8. C  TS  8TC
 TC 

72
[J] Physics (Class XI) - Volume - III

GM G  Volume  density 4 R 3 4G


9. C g 2
 2  G. 2
.  R
R R 3 R 3

4G 4G g1 R 11


g1  . 1R 1 , g 2  2 R 2  
3 3 g 2 R 22
2 3
 T1   R1 
10. A Apply Kepler’s third law,    64   
 T2   R2 
3
 R1  R1 4
  64   4    , R 1  4R 2
3

 R2  R2 1

1 GmM
11. A Total energy = Kinetic energy + Potential energy; E 0  mv 2 
2 R

mv 2 GmM GM
Centripetal force = Gravitational force;  2
 v2 
R R R

1 GM GmM GmM GmM


E0  m    P.E.    2E 0
2 R R 2R R

GM
12. D L = mvr But v 
r

Angular momentum L  m 2GMr  L r

L2 r 16r
 2   4; L2 = 4L1 = 4L
L1 r1 r

m M  m dF m 1
13. C F  G ; For maximum force, 0 or M – 2m = 0  
R2 dm M 2
2 3 2
 T1   R 1   1  1 1
3

14. C Apply Kepler’s third law, T 2  R 3 ;         


 T2   R 2   T2   2  8

T2  8  4  2  2 2 year

GM g R
15. C g 2
  2 (GM is constant) = | – 2 × 1.5 | = 3%
R g R

GM
16. C Gravitational potential at a point outside the sphere v g  . But Vg is same at a point inside the
r
hollow sphere as on the surface of sphere.

73
[J] Physics (Class XI) - Volume - III

2GM
17. B vc  ; It does not depend on mass of satellite m.; v e  m 0
R

1
18. B K.E.  Mv 02 , By Kepler’s third law; T 2  r 3 or T 2  Kr 3
2

GM
v0  where r = radius of orbit
r
1
1 GM GM 2 1 GM 2  K  3
 K.E.  M .  .   2   K.E.  T
2/3
2 r 2 r 2 T 
1 1  GM 
19. B K.E.  Mv 02  M   , r = radius of orbit
2 2  r 
E1 r2 2R  R 3R 3
   
E 2 r1 R  R 2R 2
v2 2r
20. C Centripetal acceleration  a c   ........ (i) and T  ........ (ii)
r v
Tv T 2 v2 2 T 2a c r
From (i) and (ii)  r or r 
2
; r  ;
2 4 2 4 2

T 2a c 9.32   5.26 10 


3 2

r Rh 
4 2
4 2
3
h  6.53  106  R  6.53  10 6  6.37  10 6 = 160 × 10 m = 160 km
GM GM
g  ,g
21. B
R  h
2
R2

g1 R2 g1 1
 
g  R  h 2 Given
g 100
2
1  R 
   or R + h = 10 R or h = 9 R
100  R  h 
22. D Tp = Time period of pendulum on planet
Te = Time period of pendulum on earth = 2 sec.
l l GM
 TP  2 , Te  2 , g 2
gp ge R

Tp ge G.M e R p2 2
M e R p Tp 1 2
 2   . 2;   2  2 ; Tp  2 .Te  2 2 sec
Te gp Re GM p M p R e Te 2

74
[J] Physics (Class XI) - Volume - III

2 2 2
 R  g  R  1  R  1 R
23. D g  g   ;  g      
Rh 4 Rh 4 Rh 2 Rh
R + h = 2R  h=R
24. B Angular momentum (L) = Moment of linear momentum

GM  v  GM 
 L  mv0 r  m r  
r   m GMr
0
r 

L1 r RR 1 L1 1
  1    
L2 r2 R  3R 2 L2 2

GmM
25. C P.E. of body on surface of earth = EP.  EP  
R
3 GmM 3 GmM
P.E. of body at the centre of earth =   EP  
2 R 2 R
Let v = velocity acquired by body while reaching at the centre of earth.
1 GmM 3 GmM GM 2Rg v e2
 mv 2  E P  EP    or v 
2
 Rg  
2 R 2 R R 2 2
v e2
 v2  or v  ve / 2
2
GM GM GM
 For orbital velocity, v 0   v 02  
2
26. A
r R  R 2R
1 1 GM m  gR 2 mgR mgR
 K.E.  mv 02  m  or K.E.    Kinetic energy =
2 2 2R 4R 4 4
2 3
 T   r   2.5R  R 
3

27. D  For revolving satellite, T 2  r 3   2   2   


 T1   r1   6R  R 
2
 T2   3.5R 
3
1 T1  1 24
or  T    7R   or T2    6 2 hour  T2  6 2 hour
 1    2
3
2 2 2 2
28. A

The two points are situated inside the sphere as r1 < R and r2 < R. Consider a point at distance r.
G   Mass of sphere of radius r  G  4   r 3  
F or F 
r2 r2  3
 4G  F1 r1
or F    r = (constant) r  F  r
 3  2 2

75
[J] Physics (Class XI) - Volume - III

GM
29. C Acceleration due to gravity on or near earth = g. g
R2
where R = radius of earth when g is taken on surface of earth.
GM
g when the point is near earth and r > R.
r2
Obviously r >> R.; When the earth shrinks, r and M are still the same.; Hence g remains as such
30. C

mv 2 Gm  m
Centripetal force = Gravitational force or 
 2R 
2
R

Gm 1 Gm
or v  or v 
2

4R 2 R

GMm mv 2 GMm
31. C If r < R, then F  r   r  v r
R3 r R3

GMm mv 2 GMm 1
If r > R, then F    v
r2 r r2 r

gR 2 gR 2 gR 2 g
g    
32. B At height h = R;
R  h R  R 
2 2
4R 2 4

Potential energy at surface of earth = – mgR


Potential energy at height = –mg'h
g  mgR
 P.E.   m    R  R  
4 2
 mgR  mgR mgR
 Increase potential energy =    mgR    mgR 
2 2 2
GMm
33. D Potential energy in gravitational field = E  E  Change in potential energy
r

 1 1  GMm  r2  r1 
E  GMm    
 r1 r2  r1r2
Given r1 = R, r2 = (nR + R)

GMm  nR  R  R   R g  m  nR  mgRn
2

 E   
R  nR  R  R  n  1 R  n  1
76
[J] Physics (Class XI) - Volume - III

34. C Gravitational intensity due to a mass = E


G  Mass G  100 G  10000 1 10
 E   
 distance  1  x  or x 1  x 
2 2 2
x

1
or 10 x = 1 – x or x  m
11
35. D Let volume of the bob = V  Weight of bob in air = Vg  Volume of water displaced = V
 Weight of water displaced = V × 1 × g = Vg (Here density of water = 1 g cm )
–3

 Loss in weight = Vg  Weight of bob in water = Vg  Vg


or Weight of bob in water = Vg    1 (i)
Let acceleration due to gravity when bob is in water = g'
g    1
 Weight of bob in water = mg' = Vg (ii)  Vg  Vg    1 or g 

T g  l T g  
  T  2  or 
g     1
 or T  T
Now
T g  g T    1    1
36. B

For a satellite; Centripetal force = Gravitational force


GmM
or mr   GM  R 2 g 
2
or r 32  GM or r 32  R 2 g
r2

R 2g R 2T 2g 3

 6.4 10    24  60  60 
6 2 2
 9.8
or r 3  2  where T = 24 h or r
4   3.14 
2
 4 2

or r  4.24  107 m or r = 42400 km


 Height from earth’s surface = (r – R) = 42400 – 6400 = 36000 km
2 3 2
 T2   r2   T2   6400   8 
3 3

    or           0.15 
3
37. C
 T1   r1   T1   42400   53 

or T2  T1  0.15   24   0.058  = 1.394 hour = 2 hour


3/2

Approximate time period of spy satellite = 2 hr.


77
[J] Physics (Class XI) - Volume - III

1

T g g  d 2
   1   T  d 
g  d  1 
g 1   
38. C T R ;  by binomial theorem.
T  2R 
 R

T  1  T  T 1
or  1  
T  2  6400 
 d  1km, R  6400 km or
T

2  6400

1
 Loss of time per second =
2  6400

24  60  60
 Loss in one day ie, (24 × 60 × 60) sec 
2  6400

27
 Loss per day =  7 sec approximately..
4
39. B

Gm1m 2 G4r 3 4r 3


F or F  
3   2r 
, where d = 2 r 3 2
d2

G  4 2 2 r 4
or F  or F  r 4
9

GMm  GMm  GMm


40. D Energy spent = Increase in potential energy    
3R  2R  6R

GM GM GM
Intensity, I     ..........
41. D
1  2  4
2 2 2

 
 1 1   1 
 GM 1    ..........  GM    a 
 4 16  1  Sum of series  1  r 
1  
 4

4GM 4G  3
   4G  Intensity = 4G
3 3

78
[J] Physics (Class XI) - Volume - III

GM  GM   GM 
42. B Potential V       ........
1  2   4 

 
 1 1   1 
 GM 1    .......  GM   a 
 2 4  1  Sum of series  1  r 
1  
 2

 2GM  2G 1  2G  Potentail = – 2G

dA L mvr vr
43. A Areal velocity =   
dt 2m 2m 2

dA vr r GM 1 dA
or    GMr or  r
dt 2 2 r 2 dt

44. C Centre of mass comes into picture here becasue the mass of the superdense body and the mass of the
earth are of the same order. Both will move towards their centre of mass.

Let r = distance of centre of mass from the body.

r 
 2m  0    m  H   H
2m  m 3

It indicates that the body will move towards earth through a distance H/3 under gravity.

Since H << R, the acceleration of the body is equal to acceleration due to gravity.

2 H  1  2H
 Time of journey =  S  at 2   Time 
g3  2  3g

Gm1m 2 GxM 1  x  M GM 2
45. A Gravitational force F    2 x 1  x 
r2 r2 r

d  GM 2 x 
For maximum value of force
dF
0   2 1  x   0
dx dx  r 


d
dx
 x  x 2   0  1  2x  0  x  1/ 2

79
[J] Physics (Class XI) - Volume - III

46. A

Let particle A lies at origin, particle B and C on y and x-axis respectively

 Gm m 6.67  1011  1 1 ˆ 


FAC  A
i
B ˆ
i  1.67  109 ˆi N ; Similarly F 9 ˆ
 0.2  AB  1.67  10 j N
2 2
rAB

   9
 
 Net force on particle A F  FAC  FAB  1.67  10 ˆi  ˆj N

47. C

Gmm
If two particles of mass m are placed x distance apart then force of attraction  F (Let)
x2
Now according to problem particle of mass m is placed at the centre (P) of square. Then it will experience
four forces.
Gmm
FPA = force at point P due to particle A  F
x2
G2mm G3mm G4mm
Similarly FPB  2
 2F, FPC  2
 3F and FPD   4F
x x x2
    
Hence the net force on P Fnet  FPA  FPB  FPC  FPD  2 2 F

 Gmm Gm 2
 Fnet  2 2 2 2 a
[x 
 
2
x2 a/ 2 = half of the diagonal of the square]
2

4 2 Gm 2

a2

80
[J] Physics (Class XI) - Volume - III

4 g m m R m
48. A Acceleration due to gravity g  GR  g  R or g   . R
3 e e e

gm 1 e 5 R m  g m  e  1 5
[ As g  6 and   3 ]         R m  5 R e
e m R e  g e   m  6 3 18

GM GM 4GM
We know g  R 2  D / 2 2  D 2
49. C
 
4GM 0
If mass of the planet = M0 and diameter of the planet = D0. Then g  D 02

2
GM g M R2  1  4 
50. B Acceleration due to gravity g  2  moon  moon . 2earth    
R g earth M earth R moon  80  1 

16 g
g moon  g earth  
80 5
LEVEL II
1. C

m 2R

Both the particles moves diametrically opposite position along the circular path of radius R and the
gravitational force provides required centripetal force.
mv 2 Gmm 1 Gm
 v
 2R  ;
2
R 2 R

GM GM 6.67 1011  7.34  1022


2. B We know g   R   1.87  106 m
R2 g 1.4
GM
3. C Acceleration due to gravity g 
R2
2
g planet M R  1 3
2
9
  planet  earth
     
g earth M earth  R planet
 10  1  10
If a stone is thrown with velocity u from the surface of the planet then maximum height.
u2 H planet g earth 10 10
H ; H  H planet   H earth   90  100 metre
2g earth g planet ; 9 9

81
[J] Physics (Class XI) - Volume - III

4
G  R 3
GM 3 4
4. D g 2  2
 GR
R R 3
5. A
g  R   R  1
2 2
g
      g 
g  R  h   R  2R  9 9
6. C Weight of the body at height R,
2
 
 R 
2 2
 R  2 4 4
W  W    W    W    N   72  32N
Rh R
R   3 9 9
 2
7. D

If P is the point where net gravitational force is zero then FPA  FPB

Gm1m Gm 2 m m1 d
 By solving x 
d  x  ;
2
x2 m1  m 2

m1
For the given problem d = D, m1 = earth m2 = m1 and m1 = 81 m2  m2 
81

m1 D m1 D D 9D
x   
So m1  m 2 m1 1
1 10
m1 
81 9

g 2h 100
8. B Percentage change in g when the body is raised to height h, 100%   1%
g R

g d
Percentage change in g when the body is taken into depth at,  100%   100%
g R

h
  100% (As d = h)
R

1  2h  1
 percentage decrease in weight  2  R  100   2 1%   0.5%
 
82
[J] Physics (Class XI) - Volume - III

 d g  d 3R
9. B g  g  1     g  1   d
 R 4  R 4
10. A Acceleration due to gravity at depth d,

 d  100   1 63
g  g 1    g 1    9.8 1    9.8   9.66 m / s 2
 R  6400   64  64

 d g  d d 1
11. B g  g  1   ;  g 1   ;  1
 R n  R R n

 n 1 
d R
 n 

12. A Effective acceleration due to gravity due to rotation of earth g  g  2 R cos 2 

0  g  2 R cos 2 60o

2 R 4g g 2
4
g; 
R
2
R

800
rad / sec  g  0 and   60

1
  2.5  103 rad / sec
400
13. C

m1 d
Point of zero intensity x 
m1  m 2

M
Mass of the earth m1= M; Mass of the moon m2 =
81
and distance between earth and moon d = 60 R
Point of zero intensity from the earth

M  60R 9
x   60R  54R
M 10
M
81
So distance from the moon = 60 R – 54R = 6R
83
[J] Physics (Class XI) - Volume - III

 dV ˆ V ˆ V ˆ 
14. B I  
 dx
i
dy
j k    3iˆ  4ˆj  12kˆ
dz 
  ; As V = (3x + 4y + 12z)

It is uniform field. Hence its value is same everywhere | I |  32  42  122  13 N kg 1


15. A

M = mass of earth, m0 = mass of spaceship, T = tension in string.


For spaceship
Mm 0
G 2
 T  m 0 2 R -------(1)
R
For Astronaut
GMm
 T  m2  R  l  ----------(2)
R  l 
2

F r o m ( 1 ) a n d ( 2 ) a n d u s i n g R + l = R'
GM T GM T
  
R 3
m0R  R  m R 
3

 1 1   1 1 
 T    GM  3  3 
 mR  m0 R   R R 

 GMmm0   1 1 
 T  3  3 
 m0 R  mR    R R  
This gives R   R   R   R   R   RR   R   l  3R   R 2  R 2  R R 
3 3 2 2 2

GM mm 0 3lR 2 GM 3mm 0l
 T  3 .
 m0  m  R 5 R m0  m

GM

2 Re 
3
.3ml
 m0  m  m0 


GM 3ml
g.
3 ml 3 100  200
2
.  9.8    0.01 N
R e 8R e 8 Re 8 6400 1000

84
[J] Physics (Class XI) - Volume - III

16. B Conservation of angular momentum


2 2
L  I  MR 2  = constant  T  R 2 [if M remains same]
5 T
2
T2  R 2   R / n 
2
1 24
     2; T2  hr [As T1 = 24 hr]
T1  R 1   R  n n2
17. D

Gravitational field on a mass m due to outer shell (radius r2) will be zero because the mass is placed
GM1
inside this shell. But the inner shell (radius r1) behaves like point mass placed at the centre so I 
r2
for r1  r  r2
18. C Intensity at P due to upper point = I1 and intensity at p due to lower point = I2
Net intensity at p due to spherical shell
 
I1  I 2  0  I1   I2

Gmr
19. D Intensity due to uniform circular ring at a point on its axis I 
a  r2 
2 3/2

GMmr GMm 3a
 Force on sphere F  
a 
2 3/2
 
3/2
2
r a 2  3a 
2

 
GMm 3a 3 GMm
 
 4a 
2 3/2 8a 2 [As r  3 a ]

dV
20. B As I   ; if I = 0 then V = constant
dx

K K
21. D V    E dx    3
dx  2
x 2x
22. C Gravitational intensity at point p,
GM GM
I and graviational potential; V
r2 r
Joule
 V  I  r  6N / kg  8000 km  4.8  107
kg
85
[J] Physics (Class XI) - Volume - III

h
23. C If body falls from height h then time of descent t 
g

t moon g earth
  6
t earth g moon

t moon  6 t

M m
24. D If p is the point of zero intensity, then x  d and d  x  d
M m M m

GM GM
Now potential at point p, V  V1  V2  
x dx

G M
 
2
Substituting the value of x and d – x we get V   m M
d m
25. D Applying conservation of mechanical energy between A and B point

GMm 1  GMm  1 GMm GMm


  mv 2    ; mv 2  
r 2  R  2 R r

2GM 2GM
v2    v e2  2v 02 ; v  ve2  2v02
R r

2GM GM
[As escape velocity ve  , orbital velocity v0  ]
R r
26. B When body starts falling toward earth’s surface its potential energy decreases so kinetic energy increases.
Increase in kinetic energy = Decrease in potential energy
Final kinetic energy – Initial kinetic energy = Initial potential energy – Final potential energy

 GMm   GMm   GMm   GMm 


Final kinetic energy – 0 =    =   
 r1   r2   R  h1   R  h 2 

 GMm   GMm  GMm GMm 1 GMm


     
 R  2R   R  R  3R 2R 6 R

86
[J] Physics (Class XI) - Volume - III

mgh
27. B Work done 
h
1
R
mgR 1
If h = R then work done   mgR
R 2
1
R
 GMm  GMm GM
28. B Work done  U final  U initial  U   U R  0      [As m = 1 kg]
 R  R R
29. D

Gm1m 2 Gm 2 m3 Gm1m3
Potential energy of three particles system U  
r1 2 r2 3 r1 3

Given m1 = m2 = m3 = 100 gm and r1 2  r2 3  r1 3  20 cm

 6.67 1011  101  101 


 U  3  2   1.00  1011 Joule
 20 10 

30. C When a boy jumps from a ground level upto height to then its velocity of jumping

v  2gh ----------(1)

2GM
and for the given condition this will become equal to escape velocity v escape 
R

2G  4 3 
  R d  ---------- (2)
R 3 
1/ 2
8  3 gh 
Equating (1) and (2) 2gh  R Gd ; R  
3  4 Gd 

2GM 8
31. A ve   GR 2
R 3

 v e  R if  = cosntant. Since the planet having double radius in comparison to earth therefore the
escape velocity becomes twice ie 22 m/s.

87
[J] Physics (Class XI) - Volume - III

32. C From the law of conservation of energy


Difference in potential energy between ground and maximum height = kinetic energy at the point of
projection
mgh 1 1 1
 m  kv e   mk 2 v e2  mk 2  2gR
2

h 2 [As ve  2gR ]
1 2 2
R
By solving height from the surface of earth

Rk 2
h
1 k2
33. C Kinetic energy given to rocket at the surface of earth = change in potential energy of the rocket in
reaching from ground to highest point.

1
mv 2 
mgh v2 g 1 1 2g 1 2g 1 1 2gR  v 2
     ; 
2 h ; 2 1 1
;
h R v2
;
h v2 R v2R
1 h
R h R

v2 R R
h ; h
2gR  v 2  2gR 
 2  1
 v 
34. C Potential energy of the body at a distance 4 Re from the surface of earth
mg R e mg R e mg R e
U   (As h = 4 Re)
1
h 1 4 5
Re

mg R e
So minimum energy required to escape the body will be
5

 Tplanet   5 
3/ 2
35. A According to Kepler’s law T  R 3/2 Tearth = 53/2 1 year  53/2 years

dA L mvr 1 2 dA
36. C    r ; Since L = mvr and v  r   r 2
dt 2m 2m 2 dt
3/2 3/2
TNeptune  R Neptune   1013 
37. C Kepler’s third law T 2  R 2      12   10 10
TSaturn  R Saturn   10 
38. A According to conservation of angular momentum
mv min rmax  mv max rmin  constant

rmin 1.6 1012


 v min  v max   60   12 m / s
rmax 8  1012

88
[J] Physics (Class XI) - Volume - III

39. D Time period doesnot depend upon the mass of satellite, it only depends upon the orbital radius
3/2
T1  r1 
3/2
 r  1
According to Kepler’s law     
T2  r2   2r  2 2
40. C According to law of conservation of angular momentum
d1v1
mv1d1  mv 2 d 2  v2 
d2

GM vB r vB 4R
41. B Orbital velocity of satellite v    A ;  ; vB = 6
r vA rB 3v R

1 1
42. B If F  then v 
Rn R n 1
1
here n = 1  v  11
 Ro
R

GM 6.67  1011  6  1024


43. A Orbital velocity v   ;
r 384000  103
v  1.02 km / sec  1 km / sec
44. D T2  R 3
T12  1.02 R 
3

T1  1.02R 
3/ 2

T  R 3/2

1.02R   R 3/2 100


3/2
 T1  T 
 Percentage decrease    100 
 T  R 3/2

1.02   1 1.03014  1
3/2

   100  0.0301 100  3%


1 1

R  h R  R 
3 3
8R R
45. B T  2  2 2
 2  4 2
GM gR g g

46. B Orbital radius of satellite rs  4rc given

3/2
T r 
 s  s    4
3/2
From Kepler’s law T  r 3/2
Tc  rc 

Ts = 8 Tc = 8 × 1 day = 8 days

89
[J] Physics (Class XI) - Volume - III

 9R 
3
r3 R R
T  2  2  2  9   27  2
3/2
47. B
gR 2 gR 2
g g

dV 14
48. B Gravitational intensity, I   0.7 Nkg 1
dx 20
Acceleration due to gravity, g = I = 0.7 N kg–1.
Work done under this field in displacing a body on a slope of 60o through a distance s
= m(g sin 60o)s

 
= 2  0.7  3 / 2  8  9.6J

1/3 1/3
 T 2 gR 2   GMT 2 
49. C From the expression h   2 
 R  h    R
 4   4 
2

GM 1
50. B Orbital velocity v   v (If r decreases then v increase)
r r

1 1
Percentage change in v  (Percentage change in r) = 1%   0.5%
2 2
 Orbital velocity increase by 0.5%.

LEVEL III
Single correct option.

m  nm nm 2
 
1. A
 
Here reduced mass m  nm m 1  n

r  GM  2r 3/2
 T  2 ; g  r 2   T
g Gnm 2
m 1  n 

 2H 
2. B Using g h  g  1  
 R 

 2H1   2H 2 
g1  g  1   and g 2  g  1  
 R   R 

 w 2  w1  mg 2  mg1

 2H 2 2H  H H 
= mg 1   1  1   2mg  1  2 
 R R  R R 

90
[J] Physics (Class XI) - Volume - III

3. B At the time of rest


GMm
Total energy = K E + P E = 0 + ......... (i)
d
1 GMm
On reaching the ground Ef  mv 2  .......... (ii)
2 R

 1 1
Equating (i) and (ii) v  2GM   
R d
4. C W = 4 × P.E. along sides + 2 × PE along diagonals

 Gm1m 2   Gm1m 2 
 4     2  
 0.2   0.2 2 

   6.67  10 11   0.12    6.67 10 11   0.12 


 4    2  
 0.2   0.2 2 
   
 1.33  10 11  0.47  10 11  1.8  10 11 J
 MA MB  2GM s
5. B P.E. at the piont of projection U  GM s      MA  MB 
d/2 d/2 d
Since KE + PE = 0 for satellite to escape
1 2GM s
 KE   PE or Ms v2   MA  MB 
2 d
4G  M A  M B 
 v2 
d

G  MA  MB 
or v  2
d
6. C

r h

1 mgh
By conservation of energy mv 2 
2  h
1  
here v  kve  k 2gR  R
and h = r – R

mg  r  R 
1
  rR
2
 m k 2gR  R
or k  or r 
2
2 1 r  R  / R r 1 k2
91
[J] Physics (Class XI) - Volume - III

7. B
dx a
x
Mdx
Mass of a small element dx of rod 
l
M dx
Gravitational force at a distance x from mass m  Gm
l x2
l a l a
GmM GmM  1 
F 
a
lx 2
dx  
l  x  a

1 GmM  a  l  a  GmM
v    
r l  a  l  a   a  l  a 

GM GM '
8. B Net field, I  2
 2
a  R
a  
 2
Where M' is the mass of the sphere taken out from the main sphere.
3
4 R 4 3 M
But M      and M  R   M 
3 2 3 8

 
 
GM GM GM  1 
Then I  2   1 
a  R
2
a2   R 
2

8 a    8 1   
9. A  2   2a  

2r 2 r
o
m 45 O 45
o
m
A C
2r

m
B

AD = CD = 2 r

Gm 2 Gm 2 1
Components of force due to A or C at D along OD  sin 45 
 
2
2r 2r 2 2

92
[J] Physics (Class XI) - Volume - III

Gm 2

Force due to B at D
 2r 
2

GMm mv 2 GM Gm 2 Gm 2 1 Gm 2 1 mv 2
 ; v ;  2  2 
r2 r r 4r 2 2r 2 2r 2 r
1/2
Grm  1 1 1  Gm  2  2 2  2 2  Gm 1  2 2 
v  2  
2
    ; v  
r 4 2 2 2 2  r  8  r  4 

Gravitational pull GMm


10. C Here pressure =  2
area r A

 4 
G  a 3  dSda
Then pressure of a small element da is given by dp   3 
a 2 dS
where dS is area of element of width d a
4
 G 2 ada
3
R
4
Integrating p   3 G  a da
2

2 2 2 2 2 2 2  a2 
p G  R  a   G R 1  2 
3 3  R 

4 3 3 GM 2  a 2 
Substituting R  M; p 1  
3 8 R 4  R 2 
11. C

F
r
 y
S

b a

y
Let  be the angle between r and ‘a’ then  1  e cos 
r
where e is the eccentricity. At r = a,   0
y y y y 2ab
  1  e and for r = b,   180o   1 e    2 or y 
a b a b ab

93
[J] Physics (Class XI) - Volume - III

GM s M e GM m M e
12. C During total eclipse; Total attraction due to sun and moon F1  r12

r22
When moon goes on the opposite side of earth.
GM s M e GM m M e
Effective force of attraction F2  r12

r22

2GM m M e
Change in force F  F1  F2 
r22

F 2GM m
Change in acceleration of earth a  M  r 2
e 2

F1  F2 GM s M e
Average force on earth, Fav  2

r12
Fav GM s
Average acceleration of earth, aav  M  r 2
e 1

2
a 2GM m r12  r  Mm
% change in acceleration   100  2
  100  2  1   100
a av r2 GM s  r2  M s
13. C Mass of single shell = sum of masses of two shells 4r 2  4r12  4r22 or r12  r22  r 2 where  is
surface density, r is radius of shell after coalesce; r1 and r2 are radii of individual shells. If V1 and V2 are
potentials at the centres of two shells then
GM1 GM V r 3 r12 9
V1    4Gr1 and V2    4Gr2  1  1  ie, 2 
r1 r2 V2 r2 4 r2 16

Then r1 : r2 : r  r1 : r2 :  r1  r2   9 :16 : 25 or r1 : r2 : r  3 : 4 : 5 ; V1 : V2 : V  3 : 4 : 5
2 2 2 2 2 2 2

14. A Let there be an element dx at a distance x from the origin. Its mass will be (a + bx2) dx.
Gm  a  bx 2  dx
Then dF 
x2
 l  l
a  bx 2  a   1 1  
F  Gm 

x2
dx  Gm   x

2
 b  dx

 Gm a     bl 
  l  
15. B
F F

 
d

94
[J] Physics (Class XI) - Volume - III

Gmms y 2Gmm s y
  
From figure, effective force  2  Fsin   2
d  y2 
3/2
d 2  y2 d 2  y2
2

d2 y 2Gmms y d2 y 2Gm
or s dt 2   2
m   3 y (taking y << d); ie, acceleration   y
d  y 
2 3/2 ; ie, 2
dt d

 2 
2
2Gm 1 2GM
where       2   3   
2 2
 the motion is SHM;
 T  d 2 d3

16. A The force of attraction between the complete sphere and mass m is

GmM GmM
F 
 2R  .......... (i)
2
4R 2

4 3
Mass of complete sphere is M  R .
3

4  R 
3

Mass of the cut out portion is m 0    .


3 2

M R 3R
Thus, m 0  . The distance between the centre of the cut out portion and mass m  2R   .
8 2 2
Hence the force of attraction between the cut out portion and mass m is

Gm 0 m G  M / 8  m GmM 2
f   
 3R / 2 
2
9R 2 / 4 4R 2 9

2F
Using (i), we get f  . Therefore, the force of attraction between the remaining part of the sphere
9
2F 7F
and mass m  F  f  F  
9 9
17. B Gravitational force between star of mass M and planet of mass m = centripetal force.

GMm mv 2 GM 2 v 2 2R
ie,  ; v2  ;  ; ie,  or T 
R 5/2 R R 3/2 T R T v

42 R 2 42 R 2 R 3/2


T2   ; T 2  R 7/2
v2 GM

95
[J] Physics (Class XI) - Volume - III

18. A r1  R  h1  R  R  2R and r2  R  h 2  R  3R  4R

1  KE 1 r2 4R 2
K.E     
r  KE 2 r1 2R 1

19. C As the masses of the body and the earth are comparable, they will move toward their centre of mass,
which remain stationary.

H
Hence the body of mass 2 M move through distance and time to reach the earth surface
3

2h 2H / 3 2H
  
g g 3g

1 T2 g1
20. D T we get; 
g T1 g2

g g g T2 g
Here g1  g ; g2  2
 2
   2
 h  R 4 T1 g/4
1   1  
 R  R
More than one correct option.

F1 GMr1 / R 3 r1
21. A, B When r  R ;  
F2 GMr2 / R 3 r2

F1 GM / r12 r22
when r > R;  
F2 GM / r22 r12
22. A, C
23. A, B, C
24. B, C, D
25. B, D

GMm mv 2

r r

v 2  GM

v  GM

2r 2r
The period T  
v GM
26. B, C

96
[J] Physics (Class XI) - Volume - III

Passage Comprehension Type.


Passage 1
27. C Intensity of gravitational field along the axis of a ring

GMmx
E
R  x2 
2 3/2

x E

28. A At the centre of the ring x = 0.

dU d2U
 0 and 0  The point is in stable equilibrium.
dx dx
Passage 2

v p ra
29. D Using conservation of Angular Momentum mv p rp  mv a ra ; v p rp  v a ra ; v  r
a p

1/2
 2GM rp 
1/2
 2GM  7600 103 
30. A va      3
 rp  ra ra   20000 10 12400 10 
3

1/2 1/2
 2  4  1014  7600  103   2  4  76  107 
 6 
   24516129
 20000 12400 10   2  124 
= 4951 m/s = 4.951 km/s
Passage 3

GM x M y M x v 2x  5
31. A 
R2 4R
1/2
2 4R  5R  4R 3/2
1/2
 4GM y    2r 
vx    ; Tx     T  v 
 5R  5  4GM y 
5GM y

GM x M y M y Vy2  5
32. C 
R2 R
Procedure as in Qn. No. 31

97
[J] Physics (Class XI) - Volume - III

Matrix-Match Type
33. A - p, r; B - p, r; C - q, r; D - p, r
34. A - q; B - p; C - r; D - q
35. A - s; B - q; C - r; D - p
36. A - q; B - r; C - s; D - p
37. A - q; B - p; q - t ; C - r ; D - r, s
i) Gravitational force due to planet or satellite is a central force, hence, torque = 0  Angular
momentum conservation.
ii) Kepler’s second law  constant areal velocity..
iii) In elliptical orbit as distance from focus changes, speed changes, hence K.E., while in circular
orbit speed remains same hence K.E.

2GM
iv) Escape velocity =
R

GM
where M = Mass of planet ; Orbital velocity =
R
38. A - q, r; B - t; C - q, s; D - p
A) Kinetic energy of a body projected from the surface of earth at large distance may be zero (body
momentarily comes to rest) or positive.
B) Gravitational potential energy of a bound system must be negative.
C) Change in potential energy of a point mass if left free to itself, with time may be zero (point mass
on horizontal surface) or negative (point mass falling freely vertically).

D) Change in areal velocity of earth as earth moves from apogee towards perigee is zero as areal
velocity remains constant according to Kepler’s second law.
Integer Answer Type
39. 8

path of imaginary comet


Sun
Earth
r'
r

Consider an imaginary comet moving along a strongly extended flat ellipse. The extreme points of
which are located on the earth’s orbit and at the centre of the sun. The semi-major axis of the orbit of
such a comet would apparently be half the semi-major axis of the earth’s orbit. So the time period of

98
[J] Physics (Class XI) - Volume - III

our imaginary comet T' according to Kepler’s law will given by

 T 
2
T2  r 
3
 3 = constant; T2
T  T   or T2 
2 2
 as r  r / 2 
 r 
3
r r 8

But we are interested not in the full period of the revolution of our imaginary planet but in half the period
which represnts the length of the comet’s flight one way (from the planet to the sun) and is equal to the
time which the planet will take to fall onto the sun

T T  2
t     T
ie, 2 2 8  8  ; ie, n = 8

 2GM  R   2GM  2  6.67 1011  6  1024


ve     9 103 m  9 mm
 2 
40. 9
 R 
;
 ve   3 10 
8 2

41. 8 If a body is projected from the surface of earth with a velocity v and reaches a height h, by conservation
of energy (relative to surface of earth)

1 mgh
mv 2 
2 1   h / R  

Here h = R = 6400 km and g = 10 m/s2

so v 2  gh ie, v  10  6400  103 = 8 km/s

42. 8 We know that for satellite motion

v0 
GM
R
g  GM 
r R  h  as g  R 2 and r  R  h 

1 1
In this problem v 0  ve  2gR [as ve  2gR ]
2 2

R 2g 1
So  2gR ; ie, 2R = h + R or h = R = 6400 km
Rh 2
(b) By conservation of energy

 GMm  1  GMm  1 1 
0   mv   
2
; v 2  2GM  
 R 2R 
[as r = R + h = R+ R = 2R]
 r  2  R 

GM
v  gR  10  6.4  106  8 km / s
R

99
[J] Physics (Class XI) - Volume - III

4 
G  R 3E E  / R E2
  
gp 3 R E E R E g p p 6 2
43. 2 gE 4 3  2  R  ;    
R p g E E 11 3
G  R pp  / R p p p
3 

vP 2g P R P  g  R 
The ratio of escape velocities is given by    P  P 
vE 2g E R E  gE   R E 

1/2
 6 2 6 2 2
      v p   11  2 km / s
 11 3 11  11 11

L 2.2 M s r12   11 M s r22 


44. 6 L A  2.2 M r  ; L B  11 M r 
2 2  
s 1 s 2 LB 11 M s r22 

 r1 11 
2
2.2  r1 
2
2.2  11 
 1    1   6  m1r1  m 2 r2 ;  
11  r2  11  2.2   r2 2.2 

Reasoning Type.
45. A
46. D
47. A
48. C

T  Re  h
3/2
49. A The time period of satellite, T  r 3/2 ;

For a satellite revolving close to surface of earth h = 0.  T  R 3/2 e . It is evident that the period of

revoltuion of a satellite depends upon its height above the earth’s surface. Greater is the height of a
satellite above the earth’s surface, greater is its period of revolution.
50. A

100

You might also like